Sei sulla pagina 1di 121

Physics Olympiad Papers

Colombian
Physics Olympiad
Problems

Science Olympiad Blog

Colombian Olympiad Physics


Training Problems
Problem 1-1
A hinge construction comprises three diamonds whose side lengths are in the proportions 3:2:1. Vertex A 3 is
shifted in the horizontal direction with velocity V constant. Determine the velocities of the
vertices A 1, A 2, B 2 at all angles of the building are straight.

Problem 1-2
In a movie screen shows the movement of a car. The radius of the front wheels of the car is r = 0.35 m and the
rear R of r = 1.5. The front wheels have N 1 = 6 radios. The camera filming the film moves at a speed of 24
frames per second.
a) Considering that the wheels of the car move without slipping, determine the minimum speed at which the
car must be moved to the front wheels appear to rotate in the screen.
b) What is the minimum number of radios N 2 which must be to the rear wheels while the front also appear to
not rotate?
For the next question considering that the number of spokes of the front and rear wheels is N 1 = N 2 = 6.
c) How fast is the car moving from left to right, to a spectator will appear that the spokes of the wheels rotate
in a counter clockwise?
Problem 1-3
Two tanks are moving on a horizontal ground so that approximate along the same line with constant speed as
shown. Each tank fires a shot at the same time in the same vertical plane of the other tank. The speed of each
projectile with respect to the tank is u 1 and u 2 and the respective angles with the horizontal 1
2. Both
shot tanks impinging upon.
a) Find the relative speed of the tanks.
b) Find the condition required for u 1, u 2 and 1, 2. Neglecting air resistance constant and assume the value of
the acceleration of gravity). What if the above condition is not satisfied?

Problem 1-4
To measure the wind velocity distribution as a function of the height, is rolled up balloon ascends so that
constant velocity. When the balloon is released, the dependency was measured elevation angle to the horizon
balloon as a function of time as shown in Fig. Assuming that the wind speed at the ground surface is zero and
the distance from the observer to the left point where the balloon is L = 1 km, determine the height of the
balloon after 7 minutes of being released and the wind speed so high.

Problem 2-1
Determine the force exerted on the wall of a wedge to slip her true body of mass m. The angle of the wedge
base
The coefficient of friction between the body and the surface of the wedge is
between the wedge and the ground.
Problem 2-2
A solid is placed on a rough plane and inclined an angle to horizon. One inextensible cord is attached to the
solid and its other end passing through a hole made in the plane. Initially, the rope is horizontal and is then
pulled slowly until the solid reaches the half circumference having described hole. Find the coefficient of
friction and the solid body plane.

Problem 2-3
A cable is placed on a cylinder so that part of it lies on a table and lying on the flat part. After the cable is
released it begins to move without friction. Find the speed of the cable after it has established a uniform
motion. The height of the table is equal to h.

Problem 2-4
Mass m A wedge is supported on a block of mass M and a stationary wall as shown in Fig. Find the acceleration
of each of the blocks. Neglect friction.

Problem 2-5
Determine the accelerations of the bodies of masses m 1, m 2 and m 3 mechanical system of FIG. Is negligible
friction between all surfaces in contact.

Problem 2-6
At the end of a table of length L and mass M is located a small block of mass m. The table can move without
friction on a horizontal surface. The coefficient of kinetic friction between the body and the table is
What
minimum speed V is necessary to communicate at once to the table so that it drops the block?

Problem 2-7
For a thread shaped path and of rectangular cross section made on the internal surface of a hollow cylinder
and slides a cube infinity. The inner radius of the cylinder is R basis. The thread pitch is h. The coefficient of
friction is l. Find the maximum speed (speed limit) reached by the ulna. The dimensions of the cube and the
thread profile are much smaller than R.

Problem 2-8
A rope is tied at one end to the lateral surface of a cylinder of radius r at its base and is wound around
this k times (k is an integer). A mass m is connected to the other end of the rope and a speed communicates
directed along the radius of the cylinder as shown in Fig. How long the rope is wrapped around the cylinder
again. The cylinder is located on a smooth horizontal surface.

Problem 2-9
A small metal sphere mass m is skewered on a vertical metal ring of radius R so that it can slide without
friction. The ball is connected to a massless spring which has one end fixed to the upper ring. The ball is
displaced to point P or so POA = 60 and then released.
If m = 1kg, k = 50 N / m, R = 50 cm and the natural length of the spring is 50 cm find:
a) The conditions to be satisfied by the quantities given above so that when the ball is released from the
point P or it reaches the point B.
b) The equilibrium positions of the ball along the ring. Point B Is stable equilibrium position?

Problem 2-10
Two bodies of mass m 1 and m 2 are connected by an inextensible string passing over a pulley. The planes in
which the bodies are formed with angles the horizontal and The body of the right is higher than the left

a distance h. After a time t starting the movement both bodies are at the same height. The coefficient of
friction between the bodies and surfaces is To determine the relationship between the mass of bodies.

Problem 2-11
In the system of the figure, the block of mass M can move without friction. At baseline the suspended body
mass m of yarn is separated from an angle the vertical and released. What is the mass of the body, if the
angle forms with the vertical thread does not change when moving the system?

Problem 2-12
Two small bodies of masses m 1 and m 2 are joined by an elastic spring constant k. This system is on a smooth
horizontal surface. At one point the bodies are released. The bodies begin to oscillate along a straight line.
a) What is the period of the oscillations?
b) How does each body position relative to the center of mass as a function of time?
c) How does the distance between bodies with time?
Problem 2-13
A body mass m is placed on another mass M, which is suspended from a spring elastic constant k, fig.9. At first
spring body depends only mass M, then both bodies are loosened. Find the maximum force exerted
by M on m.

Problem 3-1

A bucket of mass m lies on a horizontal surface. How high strength and low angle relative to the horizon than is
necessary to pull the bucket by its upper edge to overturn without slippage, if the friction coefficient is equal
to
Problem 3-2
A thin sheet of paper is pressed toward the table by a homogeneous mass m bar whose upper end is fastened
in an articulated manner. The angle between the bar and the sheet
the coefficient of friction between them
is There is friction between the table and paper. What minimum horizontal force must be applied to the
blade out?

Problem 3-3
A solid cylinder of radius R is supported by two blocks of the same height as shown in Fig. A block is stationary
and the other moves with speed V to the left. How the cylinder pressed force on the stationary block at the
time when the two blocks are separated by a distance
? Consider that at the initial time the blocks were
very close to each other, there is friction between the block and the cylinder.

Problem 3-4
A straight rod, uniform, homogeneous mass m and length L is placed on a horizontal table with its mass center
horizontal distance perpendicular to the edge as shown in Fig. The rod is released from rest in its horizontal
position and starts to rotate on the edge of the table. If the coefficient of static friction to the initiation of the
movement between the bar and the table is , find the angle with the horizontal bar starting reached before
sliding.

Problem 3-5
A homogeneous uniform circular cylinder rests on the edge of a horizontal step. The cylinder rolls out onto the
stage with a negligible initial velocity without slipping. Find the angle that the cylinder rotates before leaving
step and the angular velocity of the cylinder after it has rotated. Suppose that the effects of air resistance can
be neglected.

Problem 3-6
A homogeneous light stick length L is originally placed horizontally. This is supported so that it can rotate freely
around its center in a vertical plane. A spider falls with a vertical speed v o and lands located midway between
the center of the bottom and the top. The mass m of the spider is equal to the mass of the stick. Immediately
after landing the spider begins to run along this so that the angular velocity of the stick remains
constant. Determine the largest value of v or so that when the spider reaches the end of the stick it is vertical. It
is assumed that the spider is apparent from the stick when it is in vertical position. Draw a diagram of the path
followed by the spider. (Moment of inertia of a bar of length L with respect to a perpendicular axis and passing
2/12).
through its center, R = mL
Problem 3-7
A homogeneous sphere, solid of radius R and mass m is dropped. Its center of mass is at rest, but the sphere is
rotating about a horizontal axis barycentric angular velocity o. The lowest point of the sphere is at a
height h from the ground as shown in Fig. When the field is released and hits the floor it bounces a fraction
of its original height. The ball and the floor are made of materials such that the deformation of the impact both
are considered negligible. Assume that the coefficient of kinetic friction between the ball and the floor is It is
assumed that the air resistance is negligible and that the time of impact is very small but finite. (Moment of
2/5).
inertia of the sphere about an axis passing through its center, R = 2Mr
1. Assuming that the sphere moves during the impact. Find:
a) The rebound tangent angle
b) The horizontal distance traveled by the center of mass after the first impact and after the second impact.
c) The minimum possible value of

o.

2. Assume now that the slide ceases before the end of the time interval of impact. Reply back questions a) and
b).
3. Taking into account the cases 1 and 2 to graph as versus

o.

Problem 4-1
The system of figure consists of two masses m and M together by a light rod of length L. The coefficient of
friction between the rod and the edge of the step is . There is friction between the mass mand the wall. Find
an expression relating the quantities m, M, L and so that the rod remains in equilibrium on the step.

Problem 4-2
Two wedges equal in shape of isosceles triangles, each of mass m 2 and the vertical angle equal
are placed
one after the other with their bases on a horizontal plane rough. The edges of the wedges are in contact. A
mass m smooth sphere of radius r 1 and is positioned between the wedges so that the system is in
equilibrium. Prove that there is balance is met:

where is the coefficient of friction and a is the length of each wedge base.
Problem 4-3

Two identical blocks parallelepiped (dimensions a, b and c) are arranged as shown in Fig. The mass of the
blocks is m and the coefficient of friction between them and the floor is Finding the possible values of the
angle so that the system remains in the equilibrium position shown.

Problem 4-4
Three identical cylinders are positioned as shown in Fig. The coefficient of friction between the cylinders
is 1 and between the cylinders and the floor is 2. Find the condition to be satisfied 1 and 2 sothat the system
remains in balance.

Problem 4-5
Four smooth and equal spheres are placed at the bottom of a hemisphere of radius a. The radius of each
sphere is b. A fifth sphere of the same radius and density is placed on them under what conditions there is a
balance? There is no contact friction between the bodies.
Problem 4-6
Three identical inelastic cords are fixed equidistant from each other a ring of radius r and similarly to a ring of
radius 2r. The cords pass through a third ring of radius r as shown in Fig. The ring 1 is fixed in a horizontal plane
and the whole system is in equilibrium. Find the distance between the centers of the rings 2 and 3. All rings are
made of the same wire.

Problem 4-7

Pliers consist of two identical rigid parts fastened with a pivot at point O. What is the force exerted on the
pivot if the edges of the clips are pressed with a force F? Assume there is no friction in the pivot.

Problem 4-8
A rope of length L =

R / 2 is placed on the surface of a fixed cylinder of radius R, with one end located in the upper part thereof as shown in Fig. Find

the acceleration of the cord as soon as it is released. Neglect friction

Problem 4-9
A rope of length L and M is suspended mass of the ends of the points O and O 'of a horizontal roof as shown in the figure below. If the distance from the
ceiling to the lowest point of the string is H what is the tension in the rope at the lowest point of this?

Problem 4-10
A small cube of mass m is placed on an incline as shown in Fig. The angle of inclination of the plane is
the cube is

= 2tan

and the coefficient of friction between the plane and

Determine the minimum horizontal force required to move the bucket.

Problem 4-11
A light bar length d is secured to the lower ends of two spring constants k 1 and k 2 respectively as shown in Fig. At a distance x from one of them in vertical
direction throw applying a slight force F. What spring constant is observed?. Graphing the results

Problem 5-1
A body of mass m 1 = 0.1 kg is suspended from a long wire. He is bound in turn one light string of length L = 0.2
m with one small sphere of mass m 2 = 0.05 kg suspended at the other end as shown in Fig. In the small sphere
below is communicated with a very short stroke velocity v or horizontally. What value v or the bodies would be
found on the same horizontal?

Problem 5-2
A boy slowly ascends a mountain covered with snow and pulling a sled by a rope as shown in Fig. The rope is
always parallel to the tangent at any point on the mountain where the child is located. The mountain top is at
a height h and a distance L from its base. What made the boy work to bring the sled to the top of the
mountain. The mass of the sled is m and the coefficient of friction on snow is.

Problem 5-3
Resting against a system that is on a smooth horizontal surface and consisting of two bodies of
mass m, coupled by a spring with spring constant k, the speed V collides true body mass thereof. The collision
is elastic. Determine the maximum elongation of the spring.

Problem 5-4

In a thermally insulated flat tube are infinite mass m with two pistons, between which there are n mols of
monatomic gas at temperature T o. At the initial instant their velocities are directed in the same direction and
is equal to V and pV, where p is a constant. To what maximum temperature will heat the gas? The plungers do
not conduct heat. Neglect the mass of gas compared to the mass of the pistons.

Training Problems

Problem 5-5
In the figure shown how high will a sandbag of mass m 2 launched with the help of a very thin board of
mass m and length L, if on the other end of the table from the height H drops a sack of mass m 1 ?

Training Problems

Problem 5-6
Two identical balls are linked through a light rope of length L and placed on a table as shown in Fig. Determine
the height of the table so that the two balls reach the ground falling at the same time, if the ball is initially at
the right edge of the table and about to fall. The friction between the table and the balls like between the
string and the edge of the table are neglected.

Problem 5-7
In the initial state the centers of two masses M and spheres with radii R 10 are separated a distance R as
indicated below. In one load is distributed evenly + Q and-Q in the second. The first area is connected to a
remote wall with a thread which supports a maximum tension T. The second area is released at one
time. Determine the velocity of the spheres after colliding if the collision is quite inelastic. The loads are not
redistributed.

Problem 5-8
A conductor bar length L and mass m of two rigid bars hanging conductive also negligible mass and
length h which are hooked on a horizontal axis. The system is in a vertical homogeneous magnetic field
intensity B. A current pulse I or passes through the bar for a short time interval Determine the maximum
inclination of the suspension bar relative to the vertical.

Problem 5-9
A satellite moves in a circular orbit of radius R around the earth R = 2 R t (R t = radius of the earth). At one
point, the satellite is launched from a ship to a distant planet. The spacecraft is launched tangentially to the
circular orbit in the direction of movement of the satellite. As a result, the satellite is in an elliptical orbit that
passes close to the earth at a point that is opposite to the launch site of spacecraft. What is the relationship
between the mass of the spacecraft and the satellite before launch? How much time elapsed from when the
ship is launched until the satellite reaches the opposite point of its orbit?
A narrow tube of constant cross-section forms a square of side L, and is in a vertical plane. The tube is filled
with two liquids - miscible equal volume and density of 1 and 2. At first, denser liquid occupying the upper
tube. At some instant the liquids are set into motion. Find the maximum speed. No friction. The acceleration of
gravity is g.

Problem 5-11
A spacecraft in flight explodes into three equal portions. A portion continues along the original line of
flight. The other two shoot out in directions forming an angle of 60 with the original path. The energy
released in the explosion is twice larger than the kinetic energy possessed probe precisely at the time of the
explosion. Determine the kinetic energy of each fragment immediately after the explosion.
Problem 5-12
Two blocks of equal masses m are joined through an elastic spring constant k, as indicated below. The upper
block is lowered so as to compress the spring a distance x or and then released. What is the maximum height of
the center of mass of the system after releasing the upper block?

Problem 5-13
In a thermally insulated flat tube are infinite two pistons with mass m and M, between which there is a
monatomic gas occupying a volume V or P or pressure. The plungers are freed. Estimate the maximum speeds of
the pistons. Underestimate gas mass compared to the mass of the pistons.

Problem 5-14
The piston of mass M, which encloses the volume V or a monatomic gas pressure P or and
temperature T or moves at speed u. Determining the temperature and gas volume during maximum
compression.The system is thermally insulated. Neglecting the heat capacities of the plunger and the
container.

Problem 5-15
On one end of a straw length L and m is mass of mass M a grasshopper. The straw is flat on a horizontal
table. What minimum speed must the grasshopper jump to reach the other end of the straw?
Problem 5-16
Three bodies of equal mass m are interconnected by a length strings as shown in Fig. Each of the bodies has
the same charge q. If one of the threads is cut what is the maximum speed acquired by each of the masses?

Problem 5-17

Two identical particle velocities u and v, and form angles with the joining line, and the distance L from each
other as shown in Fig. The charge on each particle is q. Determine the mass of the particles, it is known that
the minimum distance which is equal to a close.

Problem 5-18
A meteor moving towards a planet of mass M (along a line through the center of the planet), hitting a spinning
artificial satellite by a circular orbit of radius R around the planet. The mass of the satellite is ten times greater
than the mass of the meteor. As a result of the crash the meteorite is embedded in the satellite, which in turn
passes into a new orbit the planet whose minimum distance is R / 2. Determine the speed or the meteorite
before the crash.
Problem 5-19
Two identical disks, mass m and radius R, slide on a frictionless ice rink and go to meet with parallel
velocities v 1 and v 2, while rotating with angular velocity 1 and 2 in opposite directions. When hitting the
edges are glued and their centers are separated by a distance equal to the diameter of both disks. Determine
the linear and angular velocities after the collision set.

Problem 6-1
A homogeneous rod, cylindrical, thin and of length L, which is rotatable about an axis O at one end, is
introduced slowly into water as indicated in the figure. The density of the rod is less than that of
water o can stay upright stick to a certain value of x, it will show as x 1. If x is smaller than x 1, a minor
perturbation of the rod is inclined as shown in Figure b. If the axis of rotation remains in the water immersing
the rod assumes a position as shown in figure c.

a) Determine the

dependence versus x.

b) Determine the value of x 1.


c) Construct a function of x, with-L <x <+ L.
Problem 6-2
A cylindrical container is filled with a fluid density and is hermetically closed by the top with a circular cap of
radius R. At the bottom of the container there is a gap S area, closed by a cork mass m.To remove the cork is to
apply a force f. What is the maximum speed that the container must have when it rotates around its vertical
axis before the cork is apparent that?

Problem 6-3
Determine the density of the alcohol, from the following elements provided.
- A test tube.
- A 500 ml beaker.
- A pipette (unscaled)
3

- Distilled water, whose density is 1 10 kg / m

3.

- Alcohol.
- Ballast (steel balls).
- A millimeter rule.
- A piece of cardboard with a hole.
- Tape
- Graph paper.
- Paper towels.
a) Describe the procedure chosen clearly indicating both the theoretical and experimental.
b) Determine the density of alcohol supplied.
c) Evaluate the error of their results.

PRECAUTIONS:
- Do not suck any liquid supplied to the mouth when using the pipette.
- In case of rupture of a glass element please notify the manager of the test.
Problem 6-4
They have two containers of the same shape, height h and attached as shown in the figure below. There is a
gap between the two containers at the height x and another of the same cross section in the vessel to the right
height and through which the water flows freely. When left container water is supplied such that this is full at
every instant of time. What is the height of the water level in the container right after it has been a very big

moment in time?
Problem 7-1
A pump piston to perform a cycle draws a volume V or air. By removing this air pump with a container of
volume V n cycles are performed. The initial vessel pressure was atmospheric pressure p o. Then, with another
pump of the same working volume V or air is introduced from the environment and also n cycles are
performed. What pressure is established in the container?
Problem 7-2
A specimen of length L is filled with hydrogen at pressure P 'is closed by a movable piston light and dipped into
a container of mercury to the depth H. What part of the length of the specimen will be filled with hydrogen
under these conditions? For what values of H has a solution this problem? The density of mercury is P and
atmospheric pressure hydrogen temperature is kept constant.

Problem 7-3
Perren scientist investigated the dependence of the number of true spherical resin particles according to the
height in the water. For particles of radius r 1 = 0.13 micrometers was obtained whose dependency graph
shown in Figure 1, (n is the concentration of the particles to the height h and No is the concentration in the
bottom of the bucket). The same graph is obtained for particles with radius r = 0.065 microns, only that the
vertical scale is magnified by a factor 8. At the same time, it is known that the density of oxygen in the
2

atmosphere decreases with altitude as shown in Figure 2


is the density of oxygen to the
height H and oxygen density in the surface of the bucket). Determining the mass of oxygen molecules. As
resin density 1.194 g / cm
or

3.

Problem 7-4
A parallelepiped-shaped container has a mass M and length L. This container is filled with a gas mass m and
also has a thin wall which slides without friction, thereby dividing the gas into two equal parts.At first gas
temperature was T. In the right part of the container connects a heater and as a result the temperature is
increased to 2T. The gas occupies the left side of the container is kept at temperature T.If the container is on a
smooth horizontal surface finding its displacement with respect to the floor.

Problem 7-5
The module of a spacecraft approaching a planet surface with constant speed along a vertical line. This
transmits data on external pressure to the module. The time dependence of pressure in arbitrary units is
shown in Fig. Data transmitted by the module after landing indicate that the outside temperature is T = 700
K and the free-fall acceleration is g = 10 m/s2. Determine:
a) the speed v at which the module falls, if the planet's atmosphere is known that carbon dioxide (CO 2)
b) the temperature T h at the height h = 15 km above the earth's surface.

Problem 8-1
A container containing a mixture of water and ice. The water mass is 10 kg. The container is brought into a
room after which the temperature of the mixture is measured immediately. The dependence of the
temperature T (t) over time t is shown in Fig. The heat capacity of water is C a = 4.2 kJ / (kgK) and the latent
heat of fusion of ice is L = 340 kJ / kg. Determining the mass m i of ice the container when it was entered into
the room. Neglecting the heat capacity of the container.

Problem 8-2
In a cylindrical vessel under a piston is 1 mol of a monatomic ideal gas. The piston mass is M, and its area
is S. What is required amount of heat to the gas communicating unit in time for the piston to move uniformly
upward velocity V? Atmospheric pressure is P o. Not considered friction between the piston and the walls.
Problem 8-3
A thermally insulated container is divided into two parts by a piston non-conductor of heat which can move
without friction by the container. On the left side of the container there is 1 mol of an ideal monatomic gas
and the right part is empty. The piston is attached to the right wall with a spring whose average length is equal
to the length of the container. To determine the relationship
for the system (heat capacity of the
system). The heat capacity of the container, the piston and the spring are neglected.

Problem 8-4
A number n of moles of gas, the volume and initial pressure is equal to V and p, is heated twice by one electric
coil which passes a current during the same time: the first constant volume V with the particularity that the
final pressure p 1, then p constant pressure starting from the same initial state, the final volume being equal
to V 2. Find the ratio = from measurements p, p 1, V 2 and V.
Problem 8-5
A vertical cylindrical vessel containing air, which is confined by a piston of mass M heavy. The air
temperature T o is equal to the temperature of the external environment. The piston is slowly raised to the
height H with respect to the equilibrium position thereof and kept there until the temperature of the air is
equal to the external environmental temperature again. Then whole thermally insulated system and the piston
is released. What will be the displacement of the piston when its oscillations end? Neglecting the heat
capacities of the container and piston neglected as the external pressure.

Problem 8-6
A thermally insulated container is divided into two parts by a non-heat conductive piston which can move
without friction by the container. In each part of the container are n moles of a gas at the same
temperature To. The vessel length is L. If left container is supplied with a quantity of heat Q, find the
distance x that as the piston moves from its equilibrium position.

Problem 8-7
With the device of figure below can find the ratio of specific heats at constant pressure and volume = C p /
C v) for a gas. It consists of two containers filled with the large volume of gas, air in this case joined by a
calibrated glass tube containing a cylindrical magnet therein sufficiently tight, appears. Using two coils placed
around the pipe, on both sides of the magnet, conveniently fed with sinusoidal alternating current, can cause
the magnet to move with simple harmonic motion of small amplitude and variable frequency and
known. Initially the switch is open. To begin the experiment the tap is closed and studied the dependence of
the amplitude and the frequency f of the oscillation of the magnet, obtaining as a result the curve in Figure b
a) Find the value of
b) Suppose the experiment is repeated, but this time, by an oversight, the key is open. Calculate the
frequency f 'self oscillating magnet
c) Under the conditions of the case b) estimate the value of the amplitude of the oscillations and m for the
frequency at which the amplitude was maximal when the key was locked.
Consider negligible friction.
n

If needed, remember that (1 + x) 1 + nx, if x

2,

The mass of the magnet is 2.0 m = g, the cross sectional area of the tube is A = 1.0 cm the volume of the area
of each side of the magnet start to oscillate before it (wrench) is V o = 0.50 L The acceleration of gravity is g =
2
10
9.8 m / s and the atmospheric pressure P o = 1.0. May Pa.
Problem 8-8
Determine the numerical value of the change in atmospheric temperature with height.
Problem 8-9
Find the change in pressure with height assuming isothermal air stratification.
Problem 8-10
Find the change in pressure, temperature, density and height atmosphere atmospheric assuming adiabatic
stratification.
Problem 8-11
In the diagram of pressure versus volume shown below the lines 1-2 and 3-4 correspond to isotherms, the
lines 2-3 and 1-4 correspond to isochoric. The line corresponds to an adiabatic 1-3. When a gas is subjected to
cycle through the path 1-2-3-1 cycle efficiency is 1. When this gas is subjected to the same cycle through the
path 1-3-4-1 cycle efficiency is 2. If this same gas undergo a cycle through the path 1-2-3-4-1, what is the
efficiency of this cycle?

Problem 8-12
The volume of a liquid to a given temperature t, is related to the volume V or t temperature or through the
relation V = V or (1 +
p) where t = t - t o and is the coefficient of volumetric expansion, which is constant
for a certain range of temperature.

The device of the figure is used to measure the coefficient of volumetric expansion of a liquid. This consists of
a glass U-tube in which a liquid is deposited. Both branches of the tube are positioned respectively within
vessels (shirts) B 1 and B 2 playing the role of thermostats. Through said sleeves is passed water at a given
temperature for each branch, thereby establishing a temperature differential between the branches of the
tube. The upper branches of the tubes, which are open to the atmosphere, is placed in third shirt B 3 at a
temperature equal to that of jacket cooler. As a result of the foregoing establishing a height difference H = h 1 h of liquid columns, which can be measured as the height H. It is evident that depends H t
2

a) According to the scheme shown what the shirt is hotter?. Explain.


b) Determine the coefficient of

expansion of the liquid as a function of

H and

t.

c) Given the result of part b), expresses the relative variation of density as a function of
graph (qualitatively) in terms of H.
HINT: (1 + x)

-1

- x, for x

and construct the

1.

d) If H = 0:
- What is the ratio between the temperatures of the sleeves B 1 and B 2?.
- What is the ratio of the densities of the liquid on shirts B 1 and B 2?.
Problem 8-13
One mole of an ideal gas is heated under such conditions that the gas pressure is proportion to its volume: V P
= where is a constant. Determine the heat capacity of the gas in such a process. Invent a device in which
the gas pressure and the volume it occupies are linked with the indicated relationship.
Problem 8-14
A cylinder closed at the top and open at the bottom, is fixed to the wall of the pool that is filled with water. On
top of cylinder are n moles of a monoatomic gas separated from water by a plunger which is the depth 2h. The
gas is heated by a resistance R causing the piston descend to a depth 3h. Find the time required to achieve
this. The resistance r is connected to a battery emf E and internal resistance negligible. Menosprciese plunger
mass, friction and thermal conductivity. The pool is wide. The density of water is and the container section
is S.

Problem 8-15
In a cylinder which holds the horizontal position to the left of the piston is an ideal gas at the right side of
cylinder vacuum has been created. The cylinder is thermally insulated from the environment while the spring
located between the plunger and the wall is initially undeformed state. The plunger is released and, after
establishing equilibrium, the volume occupied by the gas increases twice. If so, how varying the temperature
and pressure of the gas? The heat capacities of the cylinder, the piston and spring are neglected.

Problem 8-16
In a cylindrical container is in equilibrium heavy plunger heat conductor. Above the piston and below it are
equal masses identical temperature gas. The relationship between the upper and lower volume n.What will be
the ratio of the volumes if we raise the gas temperature T '?

Problem 8-17
This problem is desired to derive the equation of state of a gas in which there is a potential energy of
interaction between molecules. The way this energy is assumed to be U =
V, where is a negative
constant, N is the number of gas molecules and V is the volume occupied by the gas in the container. Similarly,

we want to find the difference between their molar heat capacities and pressure and contante volume
obtained in comparison with the case of an ideal gas.
Problem 8-18
A bulk container including piston M contains a gas at temperature T or P and atmospheric pressure or, as shown
in Fig. The piston is attached to the wall through an elastic spring constant k. Between the container and the
surface coefficient of friction exists equal to . How many times the initial temperature T should be
increased or so that the piston is located at the end of the container.?

Problem 9-1
A point particle mass m is attached to the end of a light rope of length L, which is fixed at its other end to the
tip of a cone whose axis is inclined by an angle with the horizontal. The apex of the cone forms an angle
equal 2. If the dough is slightly displaced from its equilibrium position, what is the period of small
oscillations of the mass? Neglect friction.

Problem 9-2
The system shown in the figure consists of a light rod length L and a body with mass m. One end of the rod is
embedded in the wall to a point O, so that it can rotate freely about this point in any direction. The rod is
attached to the ceiling through a string of length h light which makes an angle with the vertical and form
an
with the horizontal. Find the period of small oscillations of the mass when it is moved into the paper
(or out of paper).

Problem 9-3
A point lying on the lower generatrix of a horizontal cylindrical channel radius R, is released to the speed vo a
small ball, forming a small angle with respect to the generatrix. How often the ball will pass through the lower
generatrix of the trough length L?

Problem 9-4
At point O of a wall forming a small angle the vertical is hung by a thread of length L a ball. Then he bent the
wire with the ball at a slight angle
and released. Considering Elastic absolutely hit the ball against
the wall to find the period of the oscillations of the pendulum.

Problem 9-5
For a flat horizontal floor longitudinally slides a rod of length L thin homogeneous. The paddle speed is V o. This
comes to a large stretch of rough floor with a coefficient of friction How long it takes to stop the bar?

Problem 9-6
Hllese small oscillations period for a mass body M and q load located inside of a smooth sphere of radius R, if
the top point of the sphere is fixed leaving a charge Q.

Problem 9-7
Several experiments were conducted to examine the properties of a nonlinear resistance. First, we studied the
dependence of the resistance with temperature, and it was found that while the temperature was increased
instantly increased resistance R 1 = 50 to 100 at t 1 = 100 C (see figure below ). In another experiment,
when a constant voltage V = 60 V was applied to the resistor being the temperature was 80 C. Finally, when a
voltage V 2 = 80 V was applied to the resistor appeared spontaneous oscillations in the stream. The air
temperature in the laboratory remained constant and equal to t o = 20 C. The rate of heat loss from the
resistor is proportional to the difference between its temperature and the temperature of the
environment. The heat capacity of the resistor is C = 3 J / K.

Problem 9-8
It has a pair of identical systems movindosen on a flat horizontal surface so as to approximate constant
velocity V o. Each system comprises two identical blocks each of mass m and joined together by a light spring
with spring constant k. When the two half bodies are located at a distance L apart, the springs are
unstretched. Find the time elapsed since the central blocks distance L until again meet again at this same
distance. Assume that collisions are elastic.

Problem 9-9
A mass m is attached to the end of a horizontal rod length light L and the other end of the rod is held in a ball
joint. A vertical inextensible rope length h holds the rod at a distance from the board. Find the period of small
oscillations of the system. Neglect friction.

Problem 9-10
At the ends of a rod length of negligible mass of d = 1 m small spheres are fixed two small mass m = 1
g each. The rod is suspended so that it can rotate without friction relative to the vertical axis passing through
its center. On the same line with the rod are two large spheres of mass M = 20 kg each. The distance between
the centers of each large and small sphere is near L = 16 cm. Determine the period of small oscillations of this
pendulum rotational.

Problem 9-11
A train of very long length L constant speed V is suddenly finds a hill, which forms an
horizontal. How much time elapsed from the first car begins to rise until it comes to rest?

with the

Problem 9-12
Two parallel cylindrical rollers rotating in opposite directions. The distance between the centers of the rollers
is a. A straight bar uniform horizontal length L and weight W rests on the rollers. The coefficient of kinetic
friction between the bar and the roller is . Taking x as the distance from the center of the bar at the midpoint
between the rollers, write the equation of motion of the rod, assuming initially moves from its central position
. Finding the resultant oscillatory movement period.

Problem 9-13
A hoop of mass M and radius R is located on a cylindrical support highly rugged and radius r. The ring is
displaced slightly from its equilibrium position. Find the period of small oscillations.

Problem 9-14
A support mass M is placed on a flat table. Bracket hangs a sphere of mass m by a string of length L and
negligible mass. The yarn is separated by a small angle and released. Draw the graph of velocity versus time
support. Consider the collision between the sphere and a perfectly elastic.

Problem 10-1
Two infinite planes that intersect at an angle
divide the space in zones I, II, III and IV as shown in Fig. The
surface densities cargo planes are + and - Determine the intensity and direction of the electric field in
each of the zones.

Problem 10-2

Two identical conductive plates are positioned parallel one opposite the other at a distance d between them. If
left plate is placed a charge + Q, show that on the other plate are induced loads right - and + on the left and
right surfaces, respectively.
Problem 10-3
In a non-conductive rod bent at right angles there are two small spheres (traversed by the rod) that can move
without friction along this. The spheres each have a mass m and charge Q and-Q respectively.At baseline
spheres are at rest and are at distances d and 2 d of the right angle. If the spheres are released, how come the
time difference to the vertex spheres? Find the speed of each of the areas at the time that the separation of
the spheres is d? Neglecting the effect of gravity.

Problem 10-4
Two thin sheets of thickness d each are uniformly loaded with bulk densities - y + A negative
charge and particle mass m and reaches the positively charged plate speed v at an angle the surface of the
sheet.

a) For which values of the particle velocity will not come to the negatively charged plate.
b) How long after the point and how far will the particle A positively charged sheet in this case?
Problem 10-5
Two electric charges q are joined by two stationary walls rubber elastic cords as illustrated in Fig. The
separation of the strands caused by the interaction of the loads is much smaller than its length L. How far x will
be found? The constant of elasticity of the rubber cords is and the natural length of each cord is L.

Problem 10-6
A conductive plane has a charge + Q, and each side of it to the distances x 1 and x 2 is placed a conducting plate
parallel infinite. Find the potential difference between the plates.
Problem 10-7
Two electrons distance r from each other, with the particularity that the speed of one of them is zero velocity
and the second is directed under an acute angle to the line joining them. What will be the angle between the
velocities of the electrons when they again resulting in the distance r from each other.
Problem 10-8
Three small spheres of masses m, 2m and 5m, and charges q, q and 2q respectively are positioned along a
straight line. The spheres are at rest at the beginning and the distance of separation between neighboring
spheres is. Then, they are released. What are the final velocities of the areas where they are a great distance
apart from each other? Consider that the spheres are provided along the same line.

Problem 10-9
Two metal spheres of radii R 1 and R 2 are separated by a distance d R 1, R 1 R 2. Initially, the largest area
has a net charge of Q C. It connects to the smaller sphere initially discharged through a very thin conducting
wire. Once steady state is reached:
a) What is the net charge on each sphere?
b) Find the energy of the larger sphere before making the connection.
c) What is the energy of the system after connecting the spheres.

d) Show that the load is distributed in the spheres of radii R 1 and R 2 so that
density of electric charge.

e) Find the surface electric fields in the fields and show that these satisfy the relation
Problem 11-1

Where is the surface

In the circuit shown in Fig find load capacitor C after the circuit has been connected for a long time. Neglecting
the internal resistance of the battery.

Problem 11-2
Four identical conductive plates each area S are positioned parallel to one another and equal
distance d between them. Initially, the boards are unloaded, then the end plates are connected by a conductor
with each other and the two plates are connected to a central battery emf which is E. What is the charge that
appears on each plate?

Problem 11-3
A parallel plate capacitor A and B is constructed of square plates each 15 cm side and separated by a
distance d = 5 cm apart. The plates are in the air (e o = 8.85.10-12 F / m). A small sphere coated with
conductive paint hangs a piece of silk thread length 10 cm which hangs from point A located at the upper end
of plate A as shown in Fig. The ball of mass m = 0.1 g and radius r = 0.3 cminitially hits the plate A. Plate A is
connected to earth and the plate A is momentarily connected to a Van de Graaf generator charged to a
potential of 60,000 volts. The plates are again isolated and the ball is observed that the plate moves to B and
back to B several times until it stops being in an equilibrium position with the cord at an angle to the vertical.

a) Explain the behavior of the ball and find the value of.
b) Calculate the potential difference between the plates end.

c) Determine the number of times (K) the ball back and forth before coming to rest.
d) To a graph of the potential difference between the plates as a function of the number of twists of the ball
between the plates, V AB = f (K).
Problem 11-4
Given the following list of elements find the capacitance of a capacitor.
- A battery of unknown emf.
- A capacitor of capacitance C equal to 4.7, which is much less than the value of the unknown capacitance of
the capacitor.
- An unknown capacitance capacitor.
- A galvanometer (whose scale is not calibrated)
- Two cables.
HELP:

where r is a real number.


Problem 11-5
A capacitor of capacitance C o = 20 uF is charged to a potential difference of V o = 400 V and is connected to
another capacitor of capacitance C = 1F, which as a result is loaded. This capacitor is disconnected and the
same procedure for loading a second capacitor of the same capacitance (C = 1F), then a third and so on. Then
the capacitors are connected in series. What maximum potential difference can be obtained in this way?
Problem 6
Three parallel conductive plates are positioned relative to one another at distances d 1 and d 2. At first in the
plate 1 is a charge Q and the plates 2 and 3 are discharged. Then, the plates 2 and 3 are joined by means of a
battery voltage U, the plates 1 and 3 are connected with a driver. Finding that each load plate is set.

Problem 11-7
Consider the two situations shown in Fig. In the first situation are the plates of a capacitor with charge Q andQ separated by a distance x. The left plate is fixed. The second situation shown is similar except that the two
plates are kept at a constant potential difference E o. The problem is to find the force F required to separate
the plates a distance D x (assuming that D x << x) and suggest a method that shows how this device may be
used for measuring an electric potential difference.

Problem 12-1
Find the resistance between points a and b of the circuit of Fig. R 1 = R 5 = 1.00 W, R 2 = R 6 = 2.00 W, R 3 = R 7 =
3.00 W and R 4 = R 8 = 4.00 W .

Problem 12-2
To determine the location of damage the insulation between the conductors of a telephone bifiliar line of
length L = 4.0 km at one end of this place a source of emf E = 15 V. With this is that if the opposing ends are
separated by the battery current is I 1 = 1.0 A, and if they are joined by the battery current is I2 = 1.8A. The
resistance per unit length of conductor is r = 1.25 / km. Find the point where it is deteriorating and the
insulation resistance at that point. The battery resistance is neglected.
Problem 12-3
Find the condition to be satisfied by the resistors R 1, R 2 and R 3, so that the arrangement of the figure on the
left is electrically equivalent to the arrangement of the right figure formed by resistors r 1, r2 and r 3.

Problem 12-4
In the circuit shown in Fig a voltmeter is connected between the ends of the resistance r 1 and r 2 taking
readings of 6 volts and 4 respectively. When the voltmeter is placed between points A and B it
shows 12V. What are the real voltage drops in r 1 and r 2 ? Neglect the internal resistance of the source in the
circuit.

Problem 12-5
For the diode D 2 of the circuit shown in Fig a current I 2 = 2 mA. The characteristics of current - voltage of each
of the diodes are shown in the attached figure. Finding the currents I 1, I 3 and the electromotive force of
the U or source. The resistor values are: R 1 = 100 W and R 2 = 50 W.

Problem 12-6
To charge a battery electromotive force E = 12 V from a voltage source V = 5 V, is built a circuit which consists
of an inductor L 1 = H, a diode D and a switch K, which opens and closes periodically at intervals of
time t 1 = t 2 = 0.01 s. Determine the average current I p that load to the battery.

Problem 12-7
In the circuit of Fig all voltmeters are equal. The emf of the battery is 5 V and internal resistance is negligible. If
the voltmeter indicates higher V 1 = 2.0 V What is the reading of the other two voltmeters?

Problem 12-8
In the scheme shown in the figure, the emf of the battery E, its internal resistance r is the external
resistance R. By closing the switch K 1 galvanometer needle tilts to an angle. What angle tilts the galvanometer
needle, to be closed if K 1 K 2 is also closed? The inclination of the needle is proportional to the charge passing
through the galvanometer.

Problem 12-9
The configuration of the figure consists of constant cross-section wire. The side of the larger square is one
meter long and wire has resistance r. Determining the resistance between points A and B.

Problem 12-10
Four equal ammeters and a resistor R are connected as shown in Fig. The ammeter A 1 indicates a current I 1 =
2 A, the ammeter A 2 a current I 2 = 3. What current flows through the ammeter A 3, A4 and resistor R? Find
the relationship (r = internal resistance of the ammeter).

Problem 12-11
Two resistors R 1 and R 2 are connected in series to a source of emf E. as indicated in Fig. Find an expression to
calculate the theoretical voltage drops in the resistors R 1 and R 2 from experimental measurements with a
voltmeter of internal resistance R. Neglecting the internal resistance of the source.

Problem 12-12
50 voltmeters are connected equal and different ammeters 50 as shown in FIG If the first reading
voltmeter V 1 = 9.6 V, the first and second ammeters are I 1 = 9.5 mA, I 2 = 9.2 mA respectively .What is the sum
of the readings of the voltages indicated by the voltmeters?

Problem 13-1
The flat plates of a capacitor with space d are positioned perpendicularly to the magnetic field intensity B as
shown in Fig. In the vicinity of the cathode is a source of slow electrons that break in different directions
toward the plates. Holding what? Capacitor voltage electrons gather at the anode?

Problem 13-2

A circular loop of wire in the radius to diameter has a bonding wire thereof, parallel to the horizontal axis OO
'which can rotate around the loop, see figure below. The mass per unit length of the loop and the bridge is the
same and equal to r. The current flowing in the loop is equal to I. The loop is in the gravity field and the
magnetic field intensity B directed parallel to the gravity field. What angle tothe vertical with respect to the
loop deviate?

Problem 13-3
The figure below shows a DC motor which comprises a conductive ring on which moves the end of the rod of
length r which is pivotable about a hinge located in the center of the ring. The whole system is in the presence
of a constant magnetic field intensity B. Determine the stationary angular speed and the current in the circuit,
if the friction force in the movable contact with the ring is F.

Problem 13-4
A ring-shaped wire has a conductive connection along a diameter. The capacitors C 1 and C 2 are connected to
the semicircles as found in the figure below. The ring is placed in a linearly increasing magnetic field
induction B (t) = B or T / T perpendicular to the ring plane. At some point the connection is removed, then the
field keeps changing. Find loads established in capacitors.

Problem 13-5
The circuit shown in the figure below comprises a battery which supplies an emf E, an inductor L, two
capacitors C 1 and C 2 and a switch K. If the switch is closed at t = 0 what is the maximum current through the
coil?

Problem 13-6
An inductor L is connected in parallel with two resistors R 1 and R 2, as indicated below. Across the windings of
the coil and perpendicular to them is a constant magnetic field as shown. If just at the time when the magnetic
field is suppressed by the resistance R 1 circulating a current I which is the heat dissipated through each of the
resistors?

Problem 13-7
Two coils, which are inductors L 1 and L 2 are connected through the switches K 1 and K 2 with a capacitor C as
shown in the figure below. At the initial instant both switches are open and the charged capacitor to a
voltage V o. First switch closes K 1, and when the voltage across the capacitor is zero, the switch
closes K 2. Determine the maximum and minimum current passing through the coil L 1 after closing the
switch K 2. The resistances of the coils are neglected.

Problem 13-8

For a non-conductive surface rolls without slipping a ring uniformly loaded mass m. After connecting an
induction magnetic field B perpendicular to the plane of the hoop, the hoop strength of pressure on the halved
surface. How fast rolling ring if its charge is q?
Problem 13-9
On two parallel metal bars forming an angle with the horizon sliding rod length and mass m b. The two parallel
bars are joined at the bottom by a capacitor C. The whole system is in a homogeneous magnetic field of
value B directed vertically upwards. Initially the sliding rod is at a distance from the bottom. Determine the
time it takes to slide the rod on the bottom. How fast will end?. Neglecting the resistance of the rods.

Problem 13-10
The circuit shown in the figure comprises a battery which supplies an emf E, an inductor L, a capacitor of
capacitance C and a switch K. If the switch is closed at t = 0 what is the maximum current through the
circuit? What is the maximum voltage that presents the capacitor?

Problem 13-11
In the circuit shown in the figure, the maximum currents hllense in the inductances L 1 and L 2 after closing
the switch K. The capacitance is C, and the initial voltage U.

Problem 14-1
In the focus of a parabolic mirror of radius R and focal length f is placed a thin black disc whose size matches
the image of the sun at the focus of the mirror. What is the highest possible value of the temperature
record? The sun is a good approximation of a black body temperature T = 6000 K. Neglect thermal conduction
to the surrounding air.
Problem 14-2

Calculate the average surface temperature of the sun and earth. Assume that the sun and the earth can be
considered as black bodies and that the energy absorbed by the earth comes from the sun only.The radius of
8
the sun is R s = 7.10 m and the average distance earth sun is R st = 1.5.1011 m. The energy per unit time
2.
absorbed by the land area is E o = 1.36.103 W / m
Problem 14-3
How sunlight spacecraft orientation relative to the sun if half of the surface of the ship is a reflective surface
and the other half is black and absorbs solar radiation completely? The ship is spherical and its center of
gravity is located at its center.
Problem 14-4
The energy emitted per unit area and time, E, of a body at a temperature T is given by the Stefan-Boltzmann
4,
-8
2.
law E = s T where s = 5.67.10 W / m It is known that the energy per unit time that reaches the earth per
2.
11
unit area is E o = 1400 W / m The earth-sun distance average is 1.5 10 m. A vessel made of iron orbit
around the earth. The melting temperature of iron is 1535 C. A measure that maximum distance from the sun
the ship should be located so that it begins to melt?
Problem 14-5
The surface of a space station is a blackened area. The surface is maintained at a temperature T = 500 K due
to the operation of technical equipment at the station. Determining the temperature T x of the surface of the
station if it is enveloped by a thin sheath of black metal almost the same radio station.
Problem 15-1
Measurements show that a semitransparent mirror allows passage of 1/5 of the intensity of light impinging on
it. If two identical mirror of this kind are placed perpendicular to the incident beam, could be expected that
this pair of mirrors dejasen spend 1/25 of the light intensity. But actually about 1/10 of the incident intensity
passes through these What is the reason for the above?
Problem 15-2
A hemispherical lens radius R = 5 cm and refractive index n = 1.52 is in the air and receives on its flat face
perpendicular one cylindrical beam which covers it completely, fig.a. Marginal rays are defined as those which
emerge tangentially to the curved surface of the lens. Paraxial rays are defined as those which impinge very
close to the optical axis of the lens.

a) Determine the maximum radius of the beam of parallel rays are refracted ball directly in the face of the lens.

b) Determining the minimum radius of the ring of rays parallel to the optical axis after impact the back curve
surface, leaving the lens parallel to the optical axis.
c) Find the distance along the optical axis between the point where the marginal rays concur and presenting
the point where paraxial rays.
d) Now a display P is positioned at a distance X from the center of the sphere, parallel to the flat surface of the
lens as shown in figure b, for x greater than the focal length determined by the paraxial rays readio of the light
spot on the screen as a function of X.
Supplementary Problems

1) "Return." Why, a spacecraft returning to Earth's atmosphere its temperature is increased? Why meteors
entering the atmosphere may even melt and even evaporate?
2) "Invent yourself". Design and somehow try to photograph physical processes that occur very quickly. Justify
the physical value of his photograph.
3) "Skein of wool". A grandmother is a skein of wool ball. How the mass depends on the diameter of the
handles of the same?
4) "Nailing tiptoe". Suppose you have to drive 1994 nails (L = 50mm, = 2.5 mm) on a wooden plank. What
kind of hammer you choose to do the job with speed and quality? (Hammer type refers to mass and length of
the handle). Consider if the board is:
a.
b.

walnut.
oak.

5) "Strange maneuver." Immediately after dropping the atomic bomb on Hiroshima on August 6, 1945,
Colonel Tibbets, commander of the B29 Flying Fortress was the following maneuver: turned 60 and
swooped.With what goal? What should complete the maneuver? Why not turn 180 degrees? Discuss your
answers.
6) "Oscillating." A light rod of length L is fastened by one end to a negligible friction hinge, and the other is
linked to a spring constant k. Determine the period of small oscillations of the shaft depending on the position
of the mass m l.

7) "Viscosity". Determine experimentally how the viscosity varies depending on the vegetal temperature.
8) "Experiences with a cassette." Measure the thickness of a cassette tape of a sound. Calculate the length of
a cassette tape of 60 minutes. Measure the speed with which the tape advances.

9) "More with a cassette." Look for the cassette 60 min. an expression showing the distance H in the figure,
depending on the length of tape that has passed to the other spool. What, should keep relationship with
respect
to R and D
d What
is
the
minimum
value
of H?

10) "leveling". The tube as shown in the figure contains water in its branches. Initially there is a difference in
the levels H and eventually equalize. Evaluate the speed-leveling for given H and temperature T = const also
given. Consider two cases:
a.
b.

In no air tube.
In the tube there is air at atmospheric pressure.

Proposed problems

1) "Two drops which solidify." Two drops of molten tin and zinc are allowed to cool slowly. It turns out that
the drop of making spherical tin and zinc defined acquires flattened edges. How can you explain this?
2) "Heat transfer". Investigate
cases: T 1 and T 2 <T 1> T 2.

heat

transfer

through

vertical

column

of

water

in

two

3) "inflate pump". Investigate the dependence of the internal pressure of one balloon to inflate (or pump
inflation) of its diameter.

4) "Rainbow". Explained by a model the formation of the rainbow.


5) "balloon". A balloon is inflated with helium or oxygen and released, it appears that this begins to rise, to
what maximum height the balloon ascends?
6) "Water Sense." How not destroy a tube carrying water, you can determine which way she runs? Assume
that you can access only mentioned tube 2m.
7) "Splash". A pebble dropped into water from a height H. What is the maximum height to which water
droplets jump?
2

8) "between a hot air pipe." A vertical tube 1m height and cross sectional area of S = 50 cm is open at both
ends. At the bottom heater is installed power P = 100 W What is the air velocity in the pipe? What is the cost
3
(in m / s) of air through the tube?
9) "Cone rolling on a table." Cone, with its apex fixed at a point lying wheel without slipping on a horizontal
surface. The cone height is h and the base radius is r. What is the total angular velocity full cone if the velocity of
the center of the base v constant?

PHYSICAL FUTURE IV TOURNAMENT 1996


Proposed problems
3

1) "A balloon and a crank." A balloon of 1000 m of volume is in equilibrium at a height of H = 300 m on the
surface of the Earth. Suddenly he sits on a stork (m = 10 kg). Does he descend the balloon? How?
2) "Volume of a whale." How to determine the volume of a whale swimming in the coasts of Greenland?
3) "Power a small motor." Determine the power of an electric motor toy. Determine also its efficiency.
4) "lunar atmosphere." Suppose that on the moon, it establishes an atmosphere with a composition similar to
the Earth. What are its parameters and properties? What happens if I get on with it?
5) "Football in the Moon." Surely the moon could be a resting place for earthlings. Describe and special
situations that might occur in the Moon compared to Earth playing a football game.
6) "fragile table." When in the center of a square table placed side by a body whose mass exceeds the
value
The four legs are broken simultaneously. Find the set of points where you can put a body mass
/2
without breaking any leg.
7) "Propose yourself." Propose a theoretical physics problem and solve it.
8) "Propose yourself". Proposes experimental physics problem and solve it.
9) "Sea optometrist." If a person looks good in the water, what type of lenses you need to look good in the
air?

10) "Flashlight, holes and screen." Take a flashlight and place it in front of black cardboard with various holes
of 1 mm, 2 mm, 3 mm, multiple holes or better experience with the holes you want. Projected onto a screen at
various distances. Say observe and explain.
11) "boiling water." Try communicating heat to boil water from above. Explain what it is.
12) "illuminating with economy." We need to save energy, make a parallel between different light sources:
common bulb, halogen lamps, mercury, sodium, fluorescent, candles, LED etc.. Mention its working principle,
its spectral quality and energy efficiency.
V FUTURE PHYSICAL CONTEST 1997
1) "Hot and cold". If fully opens cold water tap, a tub is filled in t 1 = 8 min. If placed at the exit of the key then
shower hose with filling time increases to t 2 = 14 min. When the cold water tap is closed and fully open the
hot water is filling time t 3 = 12 min, with shower hose and extends time t 4 = 18 min. How soon the tub was
filled completely open if the two keys simultaneously without shower and then shower?

2) "Leaving the Moon." What part of a spaceship in circular orbit of radius 2R L around the Moon (Moon
3
radius R L = 1.74 10 km) should be fuel, so as result of a short-ignition engine, you can leave the Earth's
gravitational field? The exit velocity of the gas jet is u = 4 km / s. The acceleration of gravity at the surface of
2.
the moon is L g = 1.64 m / s
3) "Injecting". Investigate factors that depend on the output rate of a liquid from a syringe with the
force F applied to the plunger.
4) "Curiosa water pump." In the next figure shows a device used to raise water to a height h of a
stream. K 1 and k 2 are two valves that open and close. Investigate factors that determines the amount of
water that can be stored in a given time and explain the operating principle of said device

5) "Sifting water." The container has a number of very small holes in a base and depositing nevertheless this
water to a certain height, the container empties no. To investigate this phenomenon

6) "artificial gravity." In a space station project aims to create an artificial gravity. Could be a cylinder of
radius r = 300 m, rotating with an angular velocity w about the axial shaft linked with an inertial reference
systemS.
a) What should be the angular velocity w for a body in the inner surface of the cylinder experiences a weight
on the surface of the Earth?
b) If the station an object falls from a height h = 10 m

Describe qualitatively the motion of the object in the reference system S.


Determine the time required for the object arrives at the floor, or drop point.

c) If the object is launched from a ground point with respect to the return on the floor:

Is there a time limit?


So that return to t = 2.5 s in which direction and how fast over the season should be released?

7) "Doing donuts." Have you noticed that when preparing donuts, sometimes when they are floating in oil
begin to rotate at times significantly faster. Propose a theory to explain the rotation. Experiment if it thinks fit.
8) "Freedom". Proposes interesting experimental problem and solve it.
9) "Tube" U "spinning." In a U-shaped tube are poured two immiscible liquids whose densities relate as
2:1. While the tube is stationary, the area of separation of the liquid passing through the axis of symmetry of
the tube. Whither moves and how much the surface of separation if the tube is rotated with respect to the
-1?
symmetry
axis
with
angular
velocity w =
5
s

10) "Helicopter pedal." Someone built a pedal helicopter propeller low mass and diameter d = 8 m. Can a pilot
mass M = 80 kg take off on that device?
Physical Futures Tournament VI 1998
1 "starting to fly." Early balloons or dirigibles aerostatic were inflated with hydrogen. During the First World
War these airships were an easy target as a projectile however small were burnt, thus disappeared from the
battlefield.
But one day in 1918 over the skies of London received an airship appeared many impacts without fire or fall to
the ground, had been inflated with helium. But someone said: "The density of helium is nearly twice that of
hydrogen, therefore the lift force of the balloon must be halved." Is this correct? Fundamente.

2 "A paradox." A bogie which has the shape shown in the figure is filled with water. Average pressures on the
front and rear walls are equal, since only depend on the height of the liquid column and density. The front wall
being larger than the rear area, should also be higher than the fluid force exerted forwards than backwards
and exerts itself wagon should move forwards. Where is the error of this reasoning? Fundamente.
3 "playing pool". They are fashionable billiard tables whose edges are
heated. What is the benefit? How does this benefit depends on temperature?
4 "Do not panic." Lately there has been speculation about the possibility of an
asteroid hitting earth. Describe what would happen if an asteroid 10 km in
diameter and density of the Earth collided with the earth.
5 "Resources housewives." Without breaking it is possible to determine if a chicken egg is raw or
cooked. Exposing its corresponding theory.
6 "Move granted." Sit in a chair on a fairly smooth surface. Stretching strong feet and then gently picking them
up you can move from corner to corner of the room. Apparently this violates the principle that a system can
not propel yourself and external force required to accelerate. Since no external force applied on you how does
it move? How do you resolve this paradox?
7 "Speaking of football." Estimate the speed with which you can kick a soccer ball. Will there be a maximum
limit speed with which to shoot a penalty kick?
8 "Tennis Shoes". Tennis shoes must have certain features such as allowing a rapid increase in speed or
braking sharply to a basketball player, which can be determined by the coefficients of static and kinetic friction
of such shoes. Measure friction coefficients of different tennis shoes. Do you agree with this there is the
maximum speed limit at which an athlete can move forward?
PHYSICAL VII FUTURES TOURNAMENT 1999
1. - "Pendulums Pendulums and" One way to demonstrate the rotation of the earth on itself is due to the
French Foucault. A pendulum swings on a plane. To make it oscillate but it appears that this plane rotates as
time passes. Foucault is assumed as the plane of oscillation of the pendulum that changes should be the
rotating earth. Analyze the change of direction of the plane of oscillation of a pendulum in your area as the
Earth rotates beneath it. To do this perform the following steps:

Put to swing pendulums of different lengths and observe the variations of the plane of
oscillation. Clearly state your conclusions. (Note: It is desirable that the length of the pendulum is
large, why?)
Measure the angular speed of rotation of the planes of oscillation of several pendulums. Can you
state some regularity? Describe it with their conclusions.
From previous measurements determine the angular speed of the rotation of the earth on itself.
For the above purposes what is most privileged site, the North Pole or Terrestrial Ecuador? Explain
and demonstrate.

2. - "For kids" ". Discuss (preferably in digital form) the flight of a soccer ball to be shot by a player from the
point of the twelve steps. To do this run the following:

Note many successful launches from penalty kick point and sort from various points of view such as
paths, flight times and others that present some interest for physics.
For major classifications described in the first paragraph explaining the shape of the path followed by
the ball specifically indicating the role of air therein.
Measure the flight times and set its analysis and conclusions. Note force considerations mean that the
player hits the ball, the contact time between the shoe and the ball caster, size and mass of the ball,

the friction force with the air and other amounts considered material such as whether the ball is fired
in Cartagena or in Tunja, or if the ball rotates or not.
Based on the analysis performed to determine the maximum initial speed (immediately after the
shooting) that a player can print to a soccer ball.

3. - "Human warmth". Determine how much heat energy generated by the human body in a day (in
Watts). Describe your method of scoring the theoretical solution on which it is based.
4. - "Jet fly." A bottle lies within a fly on a precision scale. Does it change the scale reading if the fly flies up or
down or sideways? Do they change depending readings if the bottle is covered or open? Normally fly flapping
fly. If the bottle fly flew by jet method (taking environmental air slowly and violently expelling) what are the
answers to the above questions?
5 "See in the dark." What is the maximum distance at which a dark night you can see the light of a match
(phosphorus) without using any instruments? The solution of this problem should consist of two parts. First we
must carry out the experience and report on the observed, and secondly to do an exhibition in which
theoretical aspects are addressed such as luminous intensity of a light source and high power that impresses
the human eye .
6. - "Arson remote" Legend has it that the Greeks, led by Archimedes set fire to the Roman ships using mirrors
focusing sunlight on them. Analyze the feasibility of both not this company but its effectiveness. Apply your
analysis to a real case: a table incinerate dry 10 kg located 10 m from the mirrors in a very sunny day.
Analyze the veracity of the story and find the probability of success of the ruse of Syracusan wise.
7. "Speed limit." When an object falls vertically its speed does not increase indefinitely but due to friction with
the air reaches a maximum speed limit known generically as speed. Measure the speed limit of a ping-pong
ball is released from rest. After fully describe the measurement method for an investigation into the relevant
theory.
8. "On your bike". Analyze the motion of a bicycle from two points of view: First, the stability
conditions. Second, the effect of air resistance on the maximum possible speed.
9. "Heavy clouds." Clouds are formed by water drops. Water is 800 times denser than air. Why then the clouds
will not fall like rocks on earth? Do not just qualitative responses, perform the relevant computations.
PHYSICAL VIII FUTURE TOURNAMENT 2000
1. - "Raindrops" is often seen in a wet roof water droplets are formed which eventually break off and fall to
the ground. Determine, experimentally and theoretically the dimensions and characteristics with drops of
water immediately before falling. What holds the drop before falling?
2. - "How long a battery." Many battery-powered household items. You often hear that the batteries were
discharged. Investigate how vary, depending on the usage time, the potential difference and current supplied
by a battery. Design and conduct appropriate experiments to test his claims (limit your experiments to A or AA
batteries).
3. - "Many photons." Albert Einstein conceived the idea of considering the light as consisting of
"photons". Find out how many photons per second standard bulb emits 100 watts of power. Clearly state the
assumptions on which it is based to calculate your answer.
4. - "A new orbit." Find the path to change the orbit of a satellite from one circle to another with minimal
energy expenditure.

5. - "Prisma in balance." Study the equilibrium conditions on the water surface of a prism of wood density 0.5
3,
g / cm the cross section is an equilateral triangle knowing that their length is greater than the side the base
of the triangle. Build the prism indicated experimental and test accordingly.
6. - "Sunrise and sunset" Some authors claim that 20 minutes later that "indeed" the sun has set on the
horizon we can still receive sunlight. Show the truth or falsity of this statement. If this were true it should not
happen we start seeing "leaving" the sun 20 minutes before sunrise the place?
7. "The proper angle." An old game of throwing a stone on the surface of a lake or river not turbulent, so that
the stone made several rebounds before finally plunging into the water. Examine how depends on the number
of rebounds in terms of launch angle and speed, weight or shape of the stone. Determine under what
conditions a stone ball can bounce. Is there a limit on the number of boards that can "optimal" stone?
8. "Like in the movies." In some cowboy movies shows that the wheels turn in the opposite direction to that
actually do, ie, the car moves to the right but it is observed that the radii of the rotating wheels counterclockwise.How do you explain this paradox? Confirm your explanation with a similar experiment.
9. - "Delicate walls." Ingniese a method for measuring the thickness of the walls of a soap bubble and take it
to the practice.
PHYSICAL IX FUTURE TOURNAMENT 2001
1. - "Aircraft Constructor" With a sheet of legal size paper builds a plane. Win which:
a.
b.
c.

make it fly farther,


do fly for longer, and
explain how and why these records were achieved.

2. - "Sinking and jump". A tennis ball or similar is kept submerged in a pool. After releasing jump out of the
water. Determine the height jumped out of the water in terms of the initial conditions (depth and other
parameters).To simplify the calculations can replace the tennis ball on a wooden bucket. Ignore
friction. Support your arguments by experiment.
3. - "From snow to snow." Would it be possible from the summit of Nevado del Ruiz see the Sierra Nevada de
Santa Marta? And from the Serrania de la Macarena to the previous two?
4. - "Hot and Cold." A cup of hot water freezes first a cold water after placing simultaneously in the fridge
freezer? Perform the experiment and explain the different circumstances observed.
5. - "Layer upon layer." When your signature stamp on a piece of paper with a No. 2 pencil you make a small
layer of graphite. Calculate the number of atoms that has the height of that layer.
6. - "To get clean" Suppose your bike does not have fenders. What is the maximum speed at which you can
move through a wet highway that is not wet ... the back of the body?
7. "Eagle Eye." Einstein postulated that light consists of photons. Can the human eye to "see" a single photon?
8. "Unity is strength". A single strand of yarn breaks easily pulling end. Two, too ... But three previously
gimped ... and four? Does it grow the force to break the strings with the number of them? Remember gimped
yarn threads.
PHYSICAL X FUTURE TOURNAMENT 2002

1. - "Smoking is not good but ...". Determine experimentally and theoretically the temperature from the lit
end of a cigarette. Note that there is great variation between the instant of breathing smoke and the normal
state.
2. - "Dinosaurs in extinction." Anachronistic and ill-mannered citizen smokes a cigarette in a room 4 mx 5 m by
3 m high where there are 20 people. How many molecules of smoke and ash how many enter each attendee
lungs?
3. - "Good winds." What are the ranges of wind speeds that occur on Earth? It is known that not only on Earth
but also in some other solar system planets there winds. What are the wind speeds on the different planets in
our solar system? What is the speed of sound on these planets?
4. - "... When it dawns." Since when the day is 24 hours? It also should be investigated since when man
divided the hour into 60 minutes and a minute into 60 seconds. Where, when and how the man knew that the
earth takes about 360 days for a spin around the sun?
5. - "Fattening or thinning." Does the earth mass remains constant or increases or decreases the contrast in
significant quantities? Describe what could happen in either case.
6. - "Warming up the ice." On a table are taken at room temperature, a bowl of water and a large plate
empty. The fridge freezer will draw two ice cubes equal, one is placed on the plate and the other in the water
container. Which of the cubes will melt first? Explain what happened using the appropriate physical principles.
7. "Indurain do not know." May ride a bike without taking the handlebars. Describe how this is possible. In
particular, explain how to proceed to describe an "S" on flat land. There is sufficient qualitative
description.Quantitatively substantiate their explanations.
8. "Astronomers in action." How are the distances between stars and the distance of the star to the Earth? In
particular, how the distances were determined Earth - Sun, Earth - Moon, and the diameters of the Moon and
the Earth?
9. "Water world." In the competitions of "diving" is used springboard for the athlete. What is the reason for
using this device? What features should have a good trampoline?
10. "Ptolemy or Copernicus?".? Definitely still and the sun is just the solar system revolves around him?
PHYSICAL X FUTURE TOURNAMENT 2003
1 - Electron Marathon. Actuating the switch that turns the bulb of a lounge shows that almost instantly begins
to illuminate the bulb, which means that there are electrons flowing through its filament. How fast electrons
move once you flip the switch? How long did the electrons that were in the terminals of the switch to get to
the
bulb?
2 - Hole or hole and what will be the minimum size (volume) of a black hole? In these conditions, how much is
the gravitational acceleration on its surface? What is the escape velocity from it? Escape velocity is the speed
that an object must be to never again fall on the surface of the celestial body that launched
3 - Experimental and theoretical. Usually happens that the smallest angle of inclination of an inclined plane, a
cylindrical rough wheel on it. But if placed together two such cylinders in the way suggested in the figure, the
case that the angle of inclination of the plane to be increased without sliding cylinders. Explain why this
happens and to determine the relationship between the physical variables involved in this. Determine the
maximum angle that can take the plane to the cylinders do not move. From the experiment to determine the
relationships
between
the
relevant
friction
coefficients.

4 - Examination for myopic and presbyopic premature. On a white card two circles are painted black radio 1
cm each, so that their centers are far 2cm. Experimentally determining the maximum distance at which the
normal human eye individually distinguishes the two circumferences. Explain the observed physical and
physiologically.
5 - Go back and play. What is the maximum speed that a football player can print to the ball? An interesting
way to find out is to experimentally determine the average speed of the soccer ball in different shots on
goal. This can be examined videos of these exciting plays to "measure" the distance traveled and the time used
to
calculate
the
average
speed.
6 - high and long jump. Knowing the temperature of the solar surface can determine the average speed of the
molecules of the bark of the sun. These speeds are enormous. Why these molecules do not escape the
sun?That is, since the molecules that make up the Sun collide violently with each other how can the sun not
only
does
not
disintegrate
but
also
retains
its
spherical
shape?
7 - up and down. Often heard saying that lightning "falls" from the clouds toward Earth. Do you have full
regard this statement? You actually raise or lower the beam? Browse from qualitative insights but physically
very accurate, the conditions of production of the rays and their characteristics.
8 - Strong and weak. A Flea, a Man and a jump Elephant virtually the same distance. Will this fact some
explanation from the physical point of view? Can it be said that the flea is physically very powerful because it
can jump almost a thousand times its own height, while the elephant is a "chicken" because it is not able to
jump,
but
the
fourth
of
his
height?
9 - Hoop dancer. Investigate, describe and explain the motion of a ring to descend vertically placed cylindrical
rod. If the cylindrical rod moves with a certain speed above the rotating ring remaining at a constant
height.Investigate
and
explain
this
mechanism.
10 - One on one. Currently is often the foil. If the paper was in fact how many aluminum atoms only fit in the
thickness of the leaf? PHYSICAL FUTURE TOURNAMENT XII 2004
1 - In April thousands of water. Determine the speed of raindrops falling on the floor on a rainy day. What
the
next
rain
this
speed
will
be
different?
2 - Water not drink ... Put the stove a pot lid as shown in the figure, nearly full of water and
wait until it boils. Determine the speed of the water vapor out of the mouth of the
teapot. Describe the method used in the measurement. Does it change the speed by
decreasing
the
amount
of
water
remaining
in
the
pot? Explain.
3 - Heat with color. Turning on a light bulb filament gets its glow. Determine the
temperature of the filament of a tungsten bulb 100 W power line connected to the house at 110 V. Also
determine the temperature of the space between the filament and glass bulb. Determine the temperature
of the glass bulb. What raises the temperature of the glass of the bulb? Determine the temperature finally a
3
room
of
50
m "completely
isolated"
and
illuminated
by
the
bulb.
4 -! Blows and fly! Figure crudely tries to show a seed produced by the famous flower
"Dandelion" (Taraxacum officinale). Each seed consists of the seed itself and lots of very thin
filaments that serve as a parachute. After observing and describing several flights of these seeds
make a study of their movement from the point of view of physics. Under what conditions seeds
rise, or fall, what acceleration do, forces acting on them, push paper, its origin and its relation to
the
weight
of
the
seed,
2.

etc.

5 - Sun, heated slowly. Intensity of solar radiation on the earth's surface is I = 1370 W / m If we expose a

long
iron
block
to
this
radiation
reach
what
temperature?
Determine the intensity of solar radiation and temperature on Saturn will reach beyond that same iron
block.
6 - Stargazers lunatics. Suppose we set up a base on the moon astronomical observation and we have the
task of describing the rise and the sunset What similarities and differences do you observe about the release
and setting of the sun as seen from the Earth? Every day we see the moon rise and set but altered by
successive transformations because phases. ? "Day by Day" from the moon as we see the Earth? Do they
depend
these
observations
of
our
location
on
the
lunar
surface?
7 - Improvising gravities. If from our land area launched a ball
vertically upward with a speed of 10 m / s, after approximately two
seconds back in our hands. If now we are located in space far from
any star, inside a cylindrical vessel of 10 m radius rotating uniformly
with a period of 6.28 s (= 2pi s) and being in the "floor" launched
vertically upwards same field with a speed of 10 m / s, how much
time after our hands again? What "high" enough? Fully describe the
"pseudo gravitational field" that is experienced within the ship and
the
motion
of
the
sphere.

8 - Is it that they lack money? Currently needed in any school or a voltmeter and an ammeter. But in 1826
when George Simon Ohm enunciated his famous law had none of these instruments. Verify, using only
instruments that could have used Ohm, Ohm's law.

1st Secondary
Notes:

Read the review and consultation if you have questions.


DO NOT put your personal data or the question paper or leaves your
solutions!, We will give you a form for that.
The conceptual part is worth 40% and 60% practical part.
Have a time of 2 hours.
Conceptual part

1)

Suggests a way to measure:


a.

the thickness of a sheet of paper

b.

the thickness of the film of a soap bubble

c.

the diameter of an atom

2)

3)
4)
1)

Suggests a way to measure:


a.

Earth radius

b.

the distance between the Sun and Earth

c.

the radius of the Sun


Can be measured along the length of a curved line? If possible how would?

What are the prefixes you know who symbolizes each?


Practical part
The astronomical distances are so great, compared to land to be used much greater length
units to more easily understand the relative distances between astronomical
objects. An astronomical unit (AU) is equal to the average distance from the Earth to the Sun,
ie around

which

one astronomical

. A parsec is the distance at


unitsubtends
an

angle
(One second). A light year is the distance that light in a
year, traveling at a speed in a vacuum,
a.

Express the distance from the Sun in tierrra parsecs and light years

b.

Expressing a light year and parsec in miles.

a.

A unit of time that is sometimes used is the microscopic physics tremolo. A tremolo

2)

equals
b.

3)

Is there more tremolos in one second than seconds in a year?

Humanity has existed for some


years while the universe is around
years. If
we take the age of the universe as equivalent to a day, how many seconds long has
mankind?
A spacecraft can travel at a speed of

a.
b.

What is its speed in light years per century?


The nearest star to us is Alpha Centauri and is for

light years away, how long the

ship would take years to get to Alpha Centauri? Discuss your answer.

2nd of Secondary
Notes:

Read the review and consultation if you have questions.


DO NOT put your personal data or the question paper or leaves your
solutions!, We will give you a form for that.
The conceptual part is worth 40% and 60% practical part.
Have a time of 2 hours.

Conceptual part
We can sort the events in time. By example, the event may precede the event b and c

1)

to continue the event, which sets a temporal ordering of events a, b, and c. Hence, there is a
sense in time that distinguishes the past from the present and the future. So time is a
vector? Explain.
2)
a.

3)

Can combine two vectors of different magnitude to give a zero resultant?, Can do three
vectors?
b.
Usually is studied addition, subtraction and multiplication of vectors. Why do you think
that is not considered vectors division, is it possible to define such an operation?
The wave motion appears almost in all branches of physics. We're all familiar with water
waves. There are also sound waves, as well as light waves, radio waves and other
electromagnetic waves. And last but not least the mechanics of atoms and subatomic particles

4)

called wave mechanics. How many types of waves you know? Which are? Give some
examples.
Treatment is suitable geometric surfaces whereas it finds and other discontinuities in the

1)

wave propagation are very large compared to the wavelength. What uses geometrical optics
waves?, You can make a geometric study using such acoustic waves or seismic waves?
Practical part
A motor boat is headed in the direction N30 E to 25 miles per hour in a place where the
current is such that the resulting motion is 30 miles per hour in the direction N50 E. Find the
current velocity (magnitude and direction)

2)
a.

A tuning fork oscillates with a frequency of 440 Hz The speed of sound in air is 350 m /
s, determine the wavelength of the sound produced.

b.

c.
d.
3)

. Find the wavelength

corresponding to the frequency of


, Which is the frequency of the red
visible spectrum.
Draw two waves specifying its wavelength and amplitude.
Obtain the relative wavelength.

Suppose that a certain concave spherical mirror has a focal length of 10 cm. Find the
location of the image for object distances
a.
25 cm
b.

1)

Light propagates in the speed vacuum

10 cm

c.
5 cm
Describes the image in each case.
2nd of Secondary
PART SOLUTIONS PRACTICE
Denoting the boat speed V B, the flow velocity by V C, and the resultant velocity of V, we have V
= V B + V C, so that V C = V - V B.

To calculate V C Note that the angle between V and


160 V, thus:

the law of sines is the angle

is

. Consequently the

direction of V C is S 42 E.
2)
A significant relationship between the wavelength
velocity

, The frequency

and the propagation

this is

We then

Once again applying the relationship between the wavelength


propagation velocity

, The frequency

and the

we have

When comparing the results of these last two examples, note the difference in the orders of magnitude when it
comes to sound waves and light waves. The relative wavelength will

3)
a.

The mirror equation is:

, Where q = 16.7 cm. The positive sign indicates that

the image is real. The increase is:

b.

c.

, Where M = -0,668 therefore reduced image size

and this inverted. And q is positive then the image is located on the front side of the mirror and
is real.
The object is located in the focal point. Q = , light rays originating from an object located at
the focal point of a reflecting mirror so that the image is formed at an infinite distance from the
mirror, ie , the rays traveling parallel to each other after reflection.
q = -10 cm, the image is virtual, ie is located behind the mirror. Its increase is M = 2, ie the
image is two times larger than the object and is standing. The negative value of q means that
the image is behind the mirror is virtual.

3rd of Secondary
Notes:

1)

Read the review and consultation if you have questions.


DO NOT put your personal data or the question paper or leaves your
solutions!, We will give you a form for that.
The conceptual part is worth 40% and 60% practical part.
Have a time of 2 hours.

Conceptual part
We are told that the acceleration of a falling object due to gravity is the same for all
bodies. However, that is something that is not met in direct observation. In everyday objects
do not fall in the air at the same speed: for example a coin and a piece of paper not reaching
the ground at the same time. It is assumed that the cause is air resistance, but are we

2)

sure?. Outside of trying to test in a vacuum, something not readily available, can you think of
a simple experiment to give evidence that the acceleration is the same for all bodies?
Explain, using the Bernoulli equation, the action of a parachute to slow the free fall.

3)

According to a famous legend, when ships Romans attacked Syracuse in 214 BC


Archimedes unless the city. Soldiers stood on the shore, each with a large mirror. At a signal,
the soldiers incendiaros the Roman fleet, reflecting the sun's rays on the ships. This story has

1)

any validity practiced? That is the trick of Archimedes might have worked?
Practical part
The angle at which the sun can be seen from Earth (angular diameter) equals
approximately

. The radius of the Earth is

of the average densities of the Earth and the Sun (1 year

. Determine the ratio


,

, V is the

volume of a balloon and R is the radius).


2)

A deposit without lid rectangular (Figure 2) moves with acceleration.

The tank is filled with water to a height h. What should be the acceleration to the water begins to overflow?

3)

A spherical lens has two convex surfaces of radii 0.80 m and 1.20 m. Its refractive index

is
. Calculate its focal length and the position of the image of a point 2.00 m from
the lens.
4)
In the last seconds of free fall, an object travels three-quarters of the way full. How long
falls and from that height?
3rd of Secondary
CONCEPTUAL PART SOLUTIONS
1)
The experiment demonstrates that the doctrine is a surprising simplicity. Take a coin or medallion, on
which you will place
a piece of paper, making sure it is flush with the edge. Lift and drop the metal object horizontally, with the
paper on top.
Observe that both, paper and metal, come together on the floor. If the acceleration of the two materials were
different, the role left behind. If
happen to think it may not be the same acceleration that keeps the piece of paper on the coin, if no air pressure,
try again
slightly bending the paper. The result will be the same!.
2)
The feat of Archimedes is entirely practicable. It was rebuilt in 1973 by a Greek engineer ordered 70
mirrors (each
it about 1.5 by 1.2 meters) held by soldiers. That concentrated sunlight on a boat anchored to a 50 meters
coast. A few seconds after the mirrors were properly focused, the ship caught fire and was eventually engulfed
by the
flames. To operate the mirrors must have been slightly concave, with focus placed on the boat.
PART SOLUTIONS PRACTICE
1) During the movement of the Earth around the Sun on the Earth's gravitational force acts
where M T is the mass of the Earth, M S is the mass of the Sun, L, the radius of the orbit and the constant G
of universal gravitation. This force comunicaa Earth centripetal acceleration
where T is the period of rotation of the Earth around the Sun
According to Newton's second law we have:

but
therefore

, Or
but on the other hand we have

and
.
Combining these two equations we finally obtain the latest

that

Now using the relation


finally obtain that

.
2)

The solution is obtained in the following figure:

3)

The following table reflects the sign convection in spherical mirrors


+
R Radio
Concave
Convex
Focus f
Convergent
Divergent
Object p
Real
Virtual
Q Picture
Real
Virtual

According to these convection of signs, we write


and
It appears that the first surface and the second concave convex seen from the object side which is the
Right. Therefore

or
The fact that f is positive indicating that it is a convergent lens. To obtain the image position employ
Equation Builder wear:

, With

f and the value obtained, which gives

or
.
Q The negative sign indicates that the image is real and is therefore left lens.
Finally, the increase is

In view of the negative sign, the image must be reversed and, as M is less than unity,
also be slightly smaller object.
4)
t = 2 seconds. H = 19.6 meters.

4th of Secondary
Notes:

Read the review and consultation if you have questions.


DO NOT put your personal data or the question paper or leaves your
solutions!, We will give you a form for that.
The conceptual part is worth 40% and 60% practical part.
Have a time of 2 hours.
Conceptual part

1)

A vessel with water was placed on the end of a table as shown in Figure 1

Will you lose balance on the table if the water surface placing a piece of wood and on the latter, a weight
so that both float on the water surface?
2)

Decrease the temperature of the room when we opened the refrigerator door up and
running? Explain.

3)

Explains what isothermal and adiabatic processes.

4)

Demonstrates the parallel axis theorem:

, Where I is the inertia around the

axis of rotation arbitrary I cm is the rotational inertia parallel axis passing through the center
of mass, Mis the total mass of the object and h is the perpendicular distance between the
axes. The two axes are parallel.

1)

Practical part
A rectangular tank with no lid (Figure 2) moves with acceleration.

The tank is filled with water to a height h. What should be the acceleration to the water begins to overflow?
2)

A rope, secured at the ends, is extended with the force f. In the middle of the rope is
fastened a small weight of mass m.

Determine the period of oscillations small weight attached (Neglect the mass of the rope and not
taking into account the force of gravity)
3)
A plano-convex lens thin transverse diameter 2r, radius of curvature R, and index of
refraction n 0 is positioned such that no air left

(N 1 = 1), and to the right another transparent medium with a refractive index n 2 1 (the convex face
side is air). In air at a distance d from the lens on the optical axis is placed a point source of
monochromatic light. Demonstrate the relationship
between the image position, which
is located at a distance f
the lens and the source position d. Consider only paraxial rays. F 1 and F 2 are respectively the focal
length of the lens in the air and to the situation
when one side is in a medium with refractive index n 2.
4th of Secondary
CONCEPTUAL PART SOLUTIONS
1)

The balance will not change, since, according to Pascal's law, the pressure on the bottom of the
container will be the same in all places.

2)

The temperature in the room increases. The quantity of heat released per unit time is equal to the
power consumed by the refrigerator, as the electrical energy is transformed into heat and finally
removed from the refrigerator heat back into the room again.
PART SOLUTIONS PRACTICE
1)

The solution is obtained in the following figure:

2)

The force F acting on the weight of the inclined position of equilibrium is

As can be

seen in the figure:

As the is small, we can consider that


mass m. Therefore

while

, Where x is the vertical distance up the

Using this relationship


3)

, We finally obtain that

If to the right of the lens would then air light rays emerging from the source S0 after it is
refracted at point S1 concentrate the

distance f1 of the lens.

In correspondence with the lens formula

(1).

We then have

When filling the space to the right with a medium of refractive index n2 image
S0 light source moves to the point S, at a distance f from the lens as
seen in Fig.
For the proof of the equality sought is needed to show that

We write the law of refraction for both situations:

0 is the angle between the incident beam to the limit "lens-air."


From the expressions obtained for small values

0,

when

sin obtain:

Because

Then

Analyzing the course of a beam of rays parallel to the optical axis moving
Main can demonstrate the same way that
,

therefore:

(2)

From equations (1) and (2) we obtain

that was what we wanted to demonstrate


REVIEW of 1 Secondary
SOLUTIONS
1.

If a measured person 1782 m , How many miles it measured?


a) 1782 Km b) 1782 Km c) MKM 1782 d) 0.0001782 Km
___
Solution

e)

Prefix used:
Do not forget the prefixes:
TAB
LE 1.1
Number
10000000000000000000000000
1000000000000000000000

Decimal Power

Prefix
Yotta
Zetta

Symbol
And
Z

Oct. 18
Oct. 15
October 12
Sept. 10
June 10
March 10
February 10
10

1000000000000000000
1000000000000000
1000000000000
1000000000
1000000
1000
100
10
1
0.1
0.01
0.001
0.000001
0.000000001
0.000000000001
0.000000000000001
0.000000000000000001
0.000000000000000000001
0.000000000000000000000001

2.

Exa
Peta
Tera
Jig
Mega
Kilo
Hecto
Deca
decision
centi
military
service
mike
nano
peak
femto
ato
zepto
yocto

E
P
T
G
M
K
H
D
d
c
m

n
p
F
to
z
and

In one experiment where the temperature was measured according to


the length of a body is obtained the following data:
TABLE 2.1
T [ 5
C]
L [m] 77.3

10

17

29

35

69

110

150

244

350

500

77.3

77.4

77.5

77.7

77.9

78.2

78.9

80.1

82.3

88.2

a) Plot in the Euclidean plane following this phenomenon.


Solution
What mattered was that the student discovers that the relationship between
these
variables is not a linear relationship (no points lie on a line)

b) Which of these two quantities is a


scalar and what is a vector?
Temperature = Scalar
Length
= Scalar
c) If the length of the body 22 C voucher (77.48 0.03) m type:
Relative error L = 0.0003872
Percent Error
= 0.03872%
Do not forget the definition of relative error and percentage error:
One result is usually expressed as:
where

3.

is the average value and

is the error in x, then:

Given two vectors A 6 units making an angle of +36 with the


axis x, B 7 units and in the negative x direction. The sum of the two
vectors and the angle of the direction of the sum vector with the axis x is:
Magnitude Sum :
4128 units
Steering angle of the vector sum:
+121
Solution

Let's draw the vectors in a Euclidean plane (Figure 3.1):

The vectors are movable, so that we can transfer notice the beginning "of the
vector B at the" end "of the vector A.
The vector sum vector will be the one that begins where the vector
begins to ends and ends where the vector B:

Magnitudes us note in this graph:

From the law of cosines

get

units.

From the law of sines:

obtain that

and finally the angle

order will be:


Do not forget that for any vector e n general we can write V = V or V, where:
V: Vector
V: Vector Magnitude (Tama no O module)
u V: Vector Unit
the elements can be seen in Figure 3.4

For example for


= S u S u S = 4128.

Task 3: A

= A or A =

6 or A, B

= B u B u B = 7, S

REVIEW of Secondary 2
SOLUTIONS
1.
The area of the parallelogram given by the vectors
A = 2 or x + 3 u and - u z
and
B =-u x + u y
+z2u
is:
a) 12.12 units 2 b) 34.03 units 2 c) 9.11 units 2 d) 4.99 units 2
Solution
There are two types of multiplication between vectors:

Dot or Scalar multiplication (which is named for the result is


a scalar and the symbol used is a dot:

Cross or vector multiplication (which is named for the result


is a vector and a cross symbol used is:
Vector Multiplication of vectors A and B is defined as a new vector
which is perpendicular to the plane formed by the vectors A and B in
the advancing direction of a right-threaded screw which is rotated
towards the vector A vector B (Figure 1.1).

The magnitude of the vector A is precisely the searched area and is


given by
| A | =
AB sin )
=
Area
(1.1)
Scalar
multiplication of
vectors A and B is
defined
as
the
amount climbing from:
A B = AB cos )
(1.2)
Where is the angle between the vector A and vector B when both are
starting from the same point (Figure 1.1).
The vector A and vector B can be written as:
A=Axi+Ayj+Azk
= A x u x + A + A and u and u z z
(1.3)
B = B x i + B and j + B z k
= B x u x + B + B and u and u z z
(1.4)
where i, j, k are the unit vectors (Figure 1.2) and A x, A, and z are the
components of the vector A (Figure 1.3).

Note: The figure 1.2 is amplified because the magnitude of the vectors i, j and k unit applies,
namely: | i | = i = 1, | j | = j = 1, ... etc.

Note: For the vector B the graph of components B x, B, B z is equivalent to Figure 1.3, which will
vary the module, the direction and sense of vector B thus also the magnitudes of its components.

AzBz

Applying the definition given by equation (1.2) yields


AB =AxBx+
A and B
+
(1.5)
If A = B then A A = AA cos (0) = AA = A 2 (the angle between the
vector A and the same vector A is zero), ie the magnitude of the
vector A is given by:
(1.6)
Likewise it is evident that the magnitude of the vector B is given by:
(1.7)
From the definition of scalar multiplication, equation (1.2), we can solve
and calculate the angle
cos 1

(A B / AB) =

Substituting this value in equation (1.1) in addition to calculating the


modules of both vectors, using equations (1.6) and (1.7) and similarly
replaced in equation (1.1) yields:
| A | = AB sin =
where finally,
Area = | A | = 11.9 square units
2.
A
simple
harmonic
oscillator
is
described
by
the
equation
Where all quantities are expressed in MKS
units. Find:
a.
Amplitude: _______ Period: ______ Frequency: _______ Initial
Phase: _____

b. The position for a time of 5 s.


c. Graphing x vs. t:
Solution
a. A wave can be expressed according to the following equation
(2.1)
where
A Amplitude [m]
Angular frequency or angular speed [rad / s]
Initial Phase [rad]
t Time [s]
x Vertical Variable Distance [m]
t + Movement Phase [rad]
it holds that:
(2.2)
where

F Frequency [1 / s = Hz (Hertz)]
(Number of repetitions per unit time)
P Period [s]
(Time at which the function repeats itself)
Thus using equation (2.1) can be calculated:
Amplitude :
A=4
and The angular frequency :

Period P can be calculated from equation (2.2)


,
and also the frequency f out of the equation (2.2):
will eventually initial phase

b.

Position for a time of 5 [s] can be calculated by simply replacing


the original equation this time:

Note: Be careful with the units. In your calculator need to enable the
option to radians (Rad) so that you find the correct result. Two other
options: degrees (Deg [o]) and Grads (Grad). The equivalence is

c.

Figure 2.1

3.

If you see lightning and hear their sound after 12 s, the distance to the
lightning that fell this is:
a) 276 m b) 6.27 m c) 2761 m d) 39756. M e) 3975.6 m f)
88.2 m
Help:
The speed of sound in air is:
Solution

where
4.

The minimum height of a vertical mirror for a person of 1.68 m height


is capable of observing their entire image is:
a) 84 cm b) 27 cm c) 61 cm d) 39 cm e) 114 cm f) 100 cm g)
______
Help: Suppose that the eyes are at 10 cm below the top of the head
Solution
Figure 4.1 shows the trajectories of rays that leave the top of the head of the
individual and his toes. These rays chosen so that enter the eye after
reflection, affect the vertical mirror in points a and b respectively. The mirror
needs to occupy only that region between these two points. Projecting rays
reaching the eye towards the place where they form the mirror image is that
this should have half the height of the individual, ie 3 ft .
Note: The height is independent of the distance at which the person is located
in the mirror.

EXAM 3 of secondary
SOLUTIONS
1.

The constant force to be exercised by a car engine 1500 Kg mass to


increase the speed of 4 Km / h to 40 Km / h in 8 s is:
a)
290 N b)
167 N c)
2005 N d) 1909 N e) 1875 N
f) ______
The variation of the kinetic energy is
a)
9920 J b)
51,167 N c)
5005 N d) 3395 N e)
415 N
f) 91,500 J
The average power of the engine is
a)
1930 W b)
99 W c)
88
505 W d) 895 W e) 11
458 W
f) ______
Solution

a) From the basic equations of kinematics


(1.1)
(1.2)
(1.3)
where:
v Speed [m / s] at any time t
v 0 Initial speed [m / s]
to Acceleration [m / s 2]
x Distance [m]
We can use the equation (1.3) to calculate the acceleration of the car:

after the 2nd Law of Newton:

(1.4)
force can be calculated:

b) variation of the kinetic energy is given by the equation


therefore

c)

2.

Power Motor average is given by

A gas molecule having a speed of 300 m / s resiliently collides against


another molecule of the same mass that was initially at rest. After the
collision, the first molecule moves at an angle of 30 with the initial
direction. The speed of the two molecules after collision is:
Speed 1:
260 [m / s]
Speed 2:
150 [m / s]
and the angle formed by the incident direction and the subsequent
movement of the target molecule is:
Angle:
60 or
Solution
For the study of collisions needed to use Act Momentum
Conservation and Act Conservation Energy. The which indicate that:
Initial Momentum = Final Momentum
(Vector Equation)

Initial
Energy = Final
Energy
equation)
The situation can be schematically in Figure 2.1

(Scalar

For the x component of the movement, we have:


(2.1)
and for the component,
because the collision is elastic can
equation conservation of kinetic energy will:

get

the

In this situation: m 1 = m 2, v 1i = 300 [m / s], 1 = 30


Making m 1 = m 2 in equations (2.1), (2.2) and (2.3) get

(2.2)
following

(2.3)

or

(2.4)
(2.5
)
(2.
6)

From this set of equations we can solve for v 1f v 2f, 2.:

From equation (2.4):


(2.7)
squaring the equation (2.7) and the equation (2.5):
(2.8)
(2.9)
Adding (2.8) + (2.9), taking into account that:
(2.10)
get

(2.11
)
Substituting equation (2.6) into equation (2.11), we obtain:
where

in equation (2.6):

finally the equation (2.5) calculated

The interesting thing is that the two molecules, after the collision, move
3.

perpendicular to each other, ie


.
What fraction of the volume of an iceberg is discovered?
a) 92.1% b) 91.0% c) 93.1% d) 87.0% e) 89.3%

f) 10.7%

Help:
,
Solution
The density of any body of mass m and volume V is:
(3.1)
The weight of iceberg is:
(3.2)

where:
P H Ice Weight [N]
m H ice mass [kg]
V H ice volume [m 3]
g Gravity [m / s 2]
Note: Weight and mass are different concepts. Let's review The 2nd
Act Mechanics :
F=ma
(3.3)
This equation is valid if and only if the mass is a constant.

The Force not a consequence of the acceleration, but


otherwise, the acceleration is a result of the force

The body will communicate acceleration all forces applied to


it (although it is possible that some of them cancel each other),
ie F indicates "the resultant of all forces."

Because Force and Acceleration are vector quantities these


are characterized not only by its numerical value (magnitude)
but also for his guidance and direction.
Taking into account these important observations, we can formulate the
2nd Law Newton as: Acceleration of a body is directly proportional to
the resultant of all the forces applied to said body and directed along the
result of the Forces.
If the mass is not constant (ie the mass of the teacher's chalk, a rocket
leaving the Earth, etc..) The 2nd law of Mechanics is given by:
(3.4)
where q = Momentum is the vector introduced by Isaac Newton in his
famous Principia (1678),
[Kg m / s]
(3.5)
Let us return to the concepts of mass m and weight W:
U sing the 2nd law of Newton (3.3) we can write
P=mg
(3.6)
where the weight of (vector) P is measured in [kg m / s 2 = N]
and mass (scalar) is measured in [kg]
For example if a person has a mass m = 66 [kg] weight in the city La
Paz is:
P = m g = (66 [Kg]) (9.775 [m / s 2])
P = 645.15 [N]
Note:
The severity in the city of La Paz voucher
9775 [m / s 2]
The gravity at sea level it
9810 [m / s 2]
Returning to the calculation of the volume fraction of an iceberg is
discovered

The calculated weight of the iceberg is:


(3.2)
and the weight of the volume V A of the seawater is displaced
(3.7)
but this weight must be equal to the weight of the ice floe (Principle
Archimedes)
Thereby
(3.8)
where

4.

This is the submerged part of the iceberg, so the overhang is 10.7%


Note: Archimedes Principle: A body wholly or partly immersed in a
fluid is pushed upward by a force that equals the weight of the fluid
displaced by said body.This force acts vertically through the center of
gravity of the body.
A straight 60 feet , Weighing 100 lb , Is supported on a wall at a point
that is 48 feet above the floor. The center of gravity of the ladder in one
third of the length from the floor. A man 160 lb halfway up the
ladder. Assuming no friction with the wall and the coefficient of static
friction between the floor and the staircase is
. How man can
climb the ladder before it starts to slip?
a)
b)
c)
d)
e)
f)
______
Help: 3.28 feet = 1 m ; 1 lb = 4448 N
Solution
First find the forces exerted by the system on the floor and on the wall.
The forces acting on the system can be seen in Figure 4.1

Where
P
PE
F1
F 1V
F 1H
F2
Data

weight man on the stairs [lb]


Ladder Weight [lb]
Force ground exerts on the ladder [lb]
vertical component of F 1 [Lb]
horizontal component of F 1 [lb]
Force wall exerts on the ladder [lb]
P =
PE=
to =
c =

160 [lb]
100 [lb]
48 [feet]
60 [feet]

It is easy to calculate b (Pythagoras):


b = 36 [feet]
The line of action of P intersects the ground at a distance b / 2 of the
wall.
The line of action of P and intersects the ground at a distance b 2/3 of the
wall.
The transnational equilibrium exists if and only if the vector sum of all
external forces acting on a body in equilibrium must be zero
F=0
(4.1)
where F is the resultant of the forces.
The rotational equilibrium exists if and only if the vector sum of all
external torques acting on a body in equilibrium must be zero:
=0
(4.2)

where is the result of the torques.


For there to balance transnational we use equation (4.1) to obtain:
(4.3)
(4.4)
For there to rotational balance, choose an axis passing through the point
of contact with the ground, and using equation (4.2) we obtain:
(4.5)
replacing data in equations (4.3), (4.4) and (4.5) we obtain F 2, F and F 1V
1H:

F 2 = 85 [lb]
1H = F F 2 = 85 [lb]
1V F = P + P E = 260 [lb]
Now let us analyze the system, taking into account the coefficient of
static friction
between the floor and the steps:
Let x be the fraction of the total length of the stair before it starts
slipping. Then the equilibrium conditions are:
(4.3)
(4.4)
(4.6)
replacing data in equations (4.3), (4.4) and (4.6) we obtain
F 2 = (120 x
+ 25) [Lb]
(4.7)
1H =
+ 25) [Lb]
(4.8)
1V F
= 260 [Lb]
(4.9)
The maximum force of static friction is given by

F (120 x

(4.1
0)
but
(4.
11)
then
(4.12)
and (4.12) = (4.8), then:
from which we can solve for x:

x = 79/120
Finally the man can climb the ladder
60 x [ft] = 39.5 [feet]
before it starts slipping.

EVIEW Secondary 4th


SOLUTIONS
1.

The minimum speed necessary for a body of mass m leaves the planet
Earth is
a) 200 Km / s b) 48 Km / s c) 3 km / s d) 0.9 Km / s e) 1700 km /
s f) 11.2 Km / s
Help: G =
,
Solution
Let's look at a few lines some fundamental concepts:
As an introduction, in 1665 Isaac Newton wrote about Gravitation "... I
started to think that gravitation could extend to the orbit
of Moon ... having compared and, consequently, the force required to
keep Moon in its orbit with the force of gravity on the surface of Earth I
found that the agreement was very close ... "
Johannes Kepler discovered in 1591 which are now known as Kepler's
three laws of planetary motion:
i. All planets move in elliptical orbits with the Sun at one
focus
ii. The line joining a planet to the Sun sweeps out equal areas
in equal times.
iii. The square of the period of any planet around the Sun is
proportional to the cube of the average distance of the planet
to the Sun
Isaac Newton discovered today known as Act Universal Gravitation :
The force between two particles having masses m 1 and m 2 and which
are separated by a distance r is an attraction acting along the line
joining the particles by size and having:
(1.1)
G is the gravitational constant and has the same value for all pairs of
particles:
(1.2)
Work Work is defined as force constants:

= F x [N m = J]

(1

.3)
Work for nonconstant forces (variables) * is defined as:
[N m = J]

(1.4

)
* See Appendix at the end of this calculator solucionario.
Theorem Variation of Energy T indicates that the work effected by a
resultant force F acting on a particle, as it moves from one point to
another is equal to the change in kinetic energy E C:
(1.5)
where:
(
1.6)
then
(1.7)
Conservative force A force is conservative if the work done by her on a
particle moving in any round trip is zero. (Eg, the force of gravity)
Nonconservative force A force is conservative if the work done by her on
a particle moving in any round trip is not zero. (Eg, frictional force)

Note: If there is no change in the kinetic energy of a particle moving in


a round trip, then

and equation (1.5)

and the resultant

force acting on the particle must be conservative. Similarly, if


,
From equation (1.5) we obtain
And at least one of the forces acting
on the system will be non-conservative.
Conservation Energy When forces are conservative total energy E of the
particle studied, remains constant.
(1.8)
where E P this is Potential Energy given by
(1.9)
relationship valid if and only if the force is conservative. For instance
if
then E P = mgh.
Equation (1.8) means that

(1.10)
Therefore, the equation (1.5) becomes:
(1.11)
or the equation (1.9):

(1.12)
Well, in this question the interesting part of the equation (1.12) will
where
(1.13)
When a particle of mass m is at a distance r from the center of Earth ,
The potential energy of the system is given by equation (1.13) as
(1.14)
If the particle and Earth are separated by an infinite distance we assign
the value zero to the potential energy at zero force this configuration,
ie

. So the equation becomes:


(1.15)

where
is the work done by the conservative force (gravity) on the
particle, as it moves from infinity to a distance r from the center of Earth.
Assuming that the particle moves towards Earth along a radial line r, the
magnitude of the gravitational force F (r) acting on the particle
(assuming that r> R T) is:
(1.1
6)
the minus sign indicates that it is an attractive force, ie a force pulling the
particle towards Earth. By Therefore we can calculate Potential
Energy of equation (1.15) as
(1.17
)
substituting Equation (1.16) into equation (1.17) and integrating *:

(1.18)
* See Appendix at the end of this calculator solucionario.
The minus sign indicates that the potential energy is negative for any
infinite distance, ie, the potential energy is zero at infinity and decreases
with decreasing separation distance.This corresponds to the fact that the
gravitational force exerted on the particle by Earth Attraction is.
Escape Velocity Now we can find the gravitational potential energy of a
particle of mass m on the surface Earth using equation (1.18):
(1.19)
where R T is the radius Earth , M T is its mass.
The amount of work required, equation (1.15), to move the body from
the surface of Earth to infinity is, using equation (1.19):
(1.20)
Taking the following data:

(1.21)
we can calculate:
(1.22)
If a projectile which is situated on the surface of Earth we give you more
energy than this, then, neglecting the resistance of the atmosphere, would
escape Earth never to return.
As this happens, its kinetic energy decreases while its potential energy
increases, but their speed is never reduced to zero. The
speed Escape v 0, so that the projectile does not return to Earth Is given
by
(1.2
3)
of which we can solve equation v 0:

2.

If we consider the planet Earth as a spherical conductor


of 6400 Km Radio capacitance is:
a) 0.00712 C b) 0.0712 F c) 0.000712 F d) 0.00000712
C e)
______
Help:
Solution

3.

From the relation


found C = 0.000712 F
When 1.00 g of water, which occupies a volume of 1.00 cc, boiling at
atmospheric pressure, becomes steam at 1671 cc. The heat of
vaporization of water is 539 cal / g at 1atm. Change in internal energy of
the system is:
a)
2000 cal b)
45,788 cal c) lime 0.0333 d) 498 cal e)
17 cal f) ______
Help:
1 cal = 4186 J
Solution
Heat of vaporization C V C is the heat required for a unit mass of
substance m pass from the liquid to gaseous state holding
constant Temperature and Pressure. This is say
(3.1)
Pressure
Pressure is one way to describe Force acting on a fluid. Is defined as the
magnitude of Normal Force per unit area.
(3.2)
Work
From equation (1.4) of this solucionario

, We can write
(3.

3)
then
(3.4)
if p is a constant

(3.5)
1st Act Thermodynamics If a system state changes from an initial
equilibrium i to a final equilibrium state f in a defined way, if the heat
absorbed by the system C, the work is done by the system and the change
in T Energy inner system is E I then:
(3.6)
Now if we replace the equations (3.1) and (3.5) in equation (3.6) we get:
(3.7)
In our case
V 2 is the volume of vapor
V 1 is the volume of liquid
Replacing data:

4.

The fact that this amount is positive means that the internal energy of the
system increases during the process. That is the 539 [cal] needed to boil
1 [g] Water 41 [cal]invested in external work of expansion and
498 [cal] as internal energy are added to the system.
Consider a solid cylinder of radius R and mass M, which rolls without
sliding down an inclined plane, the inertia of the cylinder
is
. The velocity of its center of mass when the cylinder
reaches the lower part of the plane is:
a)
b)
_________
Solution

c)

d)

e)

f)

This is a problem of rotational dynamics. Consider the following:


Rigid Body Body where the constituting particles always maintained the
same position each.
If a rigid body rotates with an angular velocity in each particle of the
rotation body has a certain amount of kinetic energy.
A particle of mass m, which is at a distance r from the rotation axis
moves in a circle of radius r with an angular speed around this axis and
has a linear speed given by
(4.1)

Therefore its kinetic energy this is


(4.2)
The kinetic energy Total body is the sum of the kinetic energies of each
of its constituent particles.
If the body is rigid then all particles have the same angular
velocity but
each
particle
may
have
a
different
radius r. Therefore Kinetic Energy Total Revolving rigid body is:

(4.
3)

where n indicates the total number of masses which form the system.
Rotational Inertia Rotational inertia, denoted by I, plays the same role
in rotational motion that performed by Mass M in the transnational
movement. Is defined as

(4.4)
Note that Rotational Inertia a body depends on the particular axis which
is rotating, as well as body shape and the way it is distributed mass.
Substituting equation (4.4) in equation (4.3) we obtain Kinetic
Energy Rotation E CR:

(4.5)
Center of Mass
In physics always treat objects as if they were mere particles have mass
but not size. Since the translational motion of each particle undergoes the
same displacement body which then any movement of a particle
represents the movement of the entire body.
Even when the body turn to move or vibrate one point in the body
called center of mass that moves in the same way you would a particle
subject to the same forces.

Position the center of mass of a set of masses that lie on a line or in a


plane or in space is given by:

Now consider the problem posed considering Act Conservation Energy :


Initially the cylinder is at rest at a height h above the floor leaning
against the upper end of the inclined plane (Figure 4.1)

When rolling down the inclined plane loses the cylinder equal to a
potential energy
(4.6
)
In its motion the cylinder must earn a kinetic energy equal to
(4.7)
where

v Linear speed of the Center of Mass


Angular speed around the center of mass

Hence Conservation of Energy we have:


(4.8)

we also know that:


equation (4.8):

and

. Then substituting in

(4.9)
v clearing where we finally obtain that:

* Appendix Calculation.
Calculation Differential calculus is an important mathematical tool for
the analysis of physical systems. It's not complicated, just use some
familiar ideas (eg and applies them to a small world
(eg d, o), infinitely small, or in the words of calculation: infinitesimal.
Let your main ideas:
Euclidean plane Locus represented by:

Variable changing, which varies, is denoted by x


The change that must be considered is the following

Delta:

is a symbol used to denote a difference between a target value

and an initial value, for example

Differential: d is equal to the delta is a delta just super small, infinitely


small, infinitesimal.
Summation
is a mathematical symbol used to denote sum, for
example is used to denote the sum of all students college or the amount
of bricks with which he has built a house, etc.
Integral: Or is a mathematical symbol used to denote sum (like ),
Which is used only to denote for example the sum of the number of
atoms of oxygen in a room, or the number of drops of water having an
ocean, etc.
Function Is a relationship between the elements (dots) set x to the
elements (points) and assembly. We use the symbol
to
denote a function.
For example in the Euclidean plane a function can be positive:

If we take two points on the x axis, these will correspond to other points
on the y axis:

We
as:

know

that Delta
,

x and Delta

and are

defined

These Deltas can be represented in the Euclidean plane:

Media Ratio Is defined as the average ratio

that in physics is precisely half the speed.

Limit If we
let eg Delta x becomes very but very small, making each horizontal line
is but not touching:
What we get finally is a point we denote by dx. Obviously at this point
corresponds dy point:

Now that the notation has been made is a limit. The threshold applied to
the mean ratio or average speed

So d remains

only very small. And

It is known as the instantaneous velocity.


And these last two equations define what is referred to as derivative
calculation.
Viewing FIGURE
C.4 Question is:
How
many
points dx between x 1 and x 2? ...
Many do not? And what happens if we add? You agree that we
obtain
Sure! Of course you do.
What you have to realize is known as the fundamental theorem of
calculus:

Well ...
Other useful rules are (without proof, but we hope you've motivated you
enough to investigate more about the physics and calculus):

where k is a constant.

REVIEW
3rd of Secondary
(20072002)
Notes: Read the review and consultation if you have questions.
DO NOT put your personal data or the question paper or leaves your
solutions!, We will give you a form for that.
The conceptual part is worth 40% and 60% practical part.
Have a time of 2 hours.

1.
2.
3.
4.

1.

2.

Conceptual part
In the street the whole day a cold drizzle falls. In the kitchen is laid much laundry. Do clothes
dry faster if you open the window?
Because the paper kite tail is placed?
As can determine the density of any stone using a dynamometer and water containers?
Maximum pressure that can be measured using two U mercury manometer connected in series
by a short tube
if each of them allows to measure pressure Pa. m?
Practical part
A hexagonal pencil was pushed along the horizontal plane as
shown in Figure 1 With what values of the friction coefficient
between the stylus and the stylus slide plane for the roll plane
without?

Figure 2 was made from the photograph taken from the


tails of smoke trailing advancing two locomotives for
railroad
rectilinear
path
with
speeds
and

. The directions of

movement of trains are marked with arrows. Find the


wind speed.

3.

Three bodies with masses m 1, m 2 and m 3 can slide along


the
frictionless
horizontal
line (figure
3). Where m 1 >> m 2 and m 3 >> m 2. Determine
the
maximum speeds of the bodies ends at baseline if they were
at rest, while the body was half the speed v. The clashes are
considered quite elastic.

SOLUTIONS
Conceptual part
1.

Both on the streets and in the kitchen with the window closed the steam is saturated.

But Street temperature is lower than in the premises. Consequently, the steam pressure in the street
is lower than in the room. So when you open the window of the kitchen
steam will come out of it on the street, due to which the steam is in the kitchen is always unsaturated.
The clothes will dry faster.
2.
Thanks to the different length of the wires going from
main line to points comet
paper, the latter is stable with respect to rotation around the axis
OO 'and O 1 O 1'.
The tail of the comet facilitate stability of paper regarding the
rotation around the vertical axis O 2 O 2 '.

3.

To determine the density of the stone is essential to know its mass m and volume V:

.
Using the dynamometer can determine the value of the body weight in air P 1 and P 2 in the water.
The difference between these values is equal to the force Archimedes acting on the stone
water (Archimedes force which
acts upon the stone in the air can be underestimated). Knowing the density of water
determine the volume dela stone:

and density:

4.
The manometer mercury U-shaped (Figure 4a) measures the soprepresin
, Or
indicates how much pressure p at the elbow
Left gauge is greater than the atmospheric pressure p 0. The limitation on the range of
the pressure values to be measured
imposed by the length of the manometer tubes. You can not measure the overpressure greater
that with which the mercury reaches
the edge of the right side (according to Figure 4a). The limit value of the overpressure measured to
the gauge in question is equal to m.

In the case of series connection of two pressure gauges (figure 4b) the overpressure
left elbow gauge 1 is greater than

in

Because the pressure p 2 in the elbow

Left gauge 2 is greater than atmospheric in the magnitude


thereby
.
The compressed air in the left elbow gauge 2 occupies the volume

where S is the sectional area of the tubes. First this air volume occupied

right elbow gauge one and the same volume in the left elbow and pressure gauge 2 This air was equal to the
Atmospheric (p 0). Assuming isothermal compressed air can be applied is the law of Boyle - Mariotte:

where
.
Multiplying the numerator and denominator on the right side of this equation by

where

As

Then

, We get

Pa.

Practical part
1.
In moving the stylus up side two forces act: the normal reaction force
plane N and the friction force F (Figure 1sol).
Because the stylus is not displaced in the vertical direction then
Friction can annotate
Consider the time "critical" when the stylus touches the plane at point A. For the
pen not

. For the magnitude of the force

returns the resultant of all the forces must pass through the center of mass of the
pencil.
Accordingly, for the center of mass must pass the resultant R of the
forces N and F. If the coefficient
friction force is large and R passes under the center of mass, the pencil will spin.
Thus the condition that the pen is not turning shall be recorded as
follows:
or
is obtained where finally
.

2.

The plume released by the locomotive at point A at time t will be run by

the wind at point C. Thereby

. Where

is the wind speed (Figure 2a.). But after a time t the locomotive will be located at
point B.
The progress of the train is equal to
, Where
is the speed of the train. It is noteworthy that the plume is oriented
along
Vector
, Or what for that matter, along the vector
.

Now it's easy to find the wind speed. We draw on an arbitrary scale the vector

. Then from the origin O of

vector
we draw from the same scale the vector
. The ends of the vectors
and
we draw straight lines
parallel to
corresponding columns of smoke (Figure 2b.)
At the point M of intersection of these lines with the point O enlasamos. In precisely selected scale
OM vector is the wind speed. In effect, the OM-vector v 1 is oriented along plume dela

dragging the first locomotive, while the OM-vector v 2, along the plume of the second. As measured by
length of the vector rule module OM will find the wind speed. Equals
3.

Collisions of the mass m 2 body with mass bodies m 1 and m 3 continue until the speed becomes the same

less than the speed of one of the bodies (M 1 or M 3). But since m 1 >> m 2 and m 3 >> m 2, momentum and energy of the body
mass m 2 is much smaller than the momentum and energy of these bodies of masses m 1 and m 3. Consequently, noting the
law
conservation of energy and momentum can not take into account the energy and momentum of the body of mass m 2
cease after collision Denoting v 3 v 1 and the velocities of the bodies of masses m 1 and m 3 after
cease collisions can be noted:

Solving these equations together and considering >> m 1 m 2 and m 3 >> m 2 found:

and

REVIEW
4th of Secondary
(20072002)
Notes: Read the review and consultation if you have questions.
DO NOT put your personal data or the question paper or leaves your
solutions!, We will give you a form for that.
The conceptual part is worth 40% and 60% practical part.
Have a time of 2 hours.

1.

Conceptual part
Determine the density of an unknown liquid. You can use: option 1: two vessels with

one liquid glass tube 80 to 100 cm. long, rulers, rubber tubes, funnels.
option 2: liquid to be analyzed, graduated cylinder, liquid with known density dynamometer.
2.
In a plane mirror is an image of a candle. What will happen to this if
between the mirror and the candle is placed parallel plate glass?
3.
In the street the whole day a cold drizzle falls. In the kitchen is laid much laundry.
Do clothes dry faster if you open the window?
4.
In a vase with water that rotates around its axis (figure 1) is
released to a ball that floats
Awash. What part of the area will be located the ball?

Practical part
1.
The foam cube with a mass M = 100 g. is situated on a horizontal
support
(Figure 2). The hub height is h = 10 cm. Below the cube is pierced by a
bullet that flies

vertically and whose mass is m = 10 gr. The speed of the bullet entry in the
bucket is

to the output

2.

. Toddle onto the cube or not?

From the South Pole and the North Pole of the Earth simultaneously take off two rockets with equal

Initial velocities directed horizontally. Within the time

rockets were

at the maximum distance from one another. Determine the maximum distance between the rockets. The acceleration of fall
on Earth is considered known. The radius of the Earth is
3.

In the space between the walls of the ampoule in a pressure flask was established

to

ambient temperature. Appreciating the time during which the content in the tea thermos be cooled
from 90

C to 70 0 C The surface of the blister is

specific heat capacity of water is

. The heat capacity is 1 liter. The


, The universal gas constant is

. Not taking into account the heat leakage through the cap.
SOLUTIONS
Conceptual part
1.
By immersing the same in two different liquids load forces acting on Archimedes
the same is determined as follows:
,
where

and

(1)

are the densities of the liquids, one of which is unknown.

Values F 1 and F 2 can be determined by the difference of the indications of the dynamometer which is
suspended load in cases when the latter is in the air and in the liquid:
,

(2)

where P Dynamometer is the indication when the load was in the air, P 1 and P 2 the indications
dynamometer, when the load is in the density liquids known and unknown.
From equations (1) and (2) we find the unknown density of the liquid:

2.

In tracing the course of a few rays is easy to be convinced that, after entering the candle

and the mirror is disposed parallel flat plate glass, the image of the candle will approach the mirror.
3.

See sec Third solutions.

4.

Since the ball floats to flower the water, the density of the material itself is lower than the density

Water
. Suppose that the ball is located at a distance R from the axis of a rotating vessel.
If the density of the ball is equal to the density of water, it would be an invariable distance
axis of rotation. The centripetal acceleration may be communicated to the resulting ball force
and gravity forces of the surrounding water pressure. In this resulting module would equal
where

is the angular speed of rotation of the container V, the volume of the pellet.

The pellet density


Arranged in the same point from the surrounding water acts a
Like force the ball now communicates acceleration

This acceleration is greater than that needed for rotation by the circle of radius R.
Accordingly, the equilibrium position of the ball is located in the axis of the container.
Practical part
1.
The cube can jump if the magnitude of the force F, which acts on the same part of the bullet,
is greater than the magnitude of the force of gravity

. Let us find this force. For this

examine the bullet. In the same from the bucket acts a force equal in magnitude, but
opposite in direction to the force of gravity m g. The speed of the bullet passing through the hub,
varies insignificant variation is equal to 5 m / s, which is only 5% of the
speed of the bullet to penetrate into the bucket. Therefore we can consider that no force F
depends on the speed of the bullet and is constant. The momentum of the bullet passing through the hub,
to effect changes in the bullet action of two forces: the force of gravity and the force
friction. If the time in which the bullet passes through the whole cube, is denoted by
,
then
(1). Time
not hard to find him.
Since the forces acting on the hub are constant, the acceleration is constant also
Bullet and therefore the speed of the bullet changes linearly with time.
Therefore the average speed of movement of the bullet in the cube is equal to
.
Accordingly, the bullet passes through the hub in time
.
Substituting this value

in Eq. (1) we find:


.

As

is small, the value of

is much smaller than the pulse variation

Bullet and can be neglected. The force F was greater than the strength of the
gravity acting on the hub. So skip the bucket.
2.
Rockets move by ellipses. The takeoff point corresponds to the distance
minimum from the center of the earth, while the point of the orbit that is encime
Earth point diametrically opposite to the apogee of the orbit. At these points the
rocket velocity is perpendicular to the line drawn from the center of the Earth to orbit.
L denote the length of the major axis of the orbit. Then the maximum distance s between
rockets will equal

. The period T of the rocket flight orbit equals

to
. If the flight period for the circular orbit of radius R t is denoted by T 1,
then according to Kepler's third law

where

.
Because centripetal acceleration of the satellite moving the circular orbit
radius R p is equal to g, then
.

Therefore

.
Thus

.
.
3.
Denote temperature T 1 and T 2 tea room temperature.
Wall bumping warm air molecules acquire contained in the ampoule
kinetic energy

. But upon contacting the cold wall kinetic energy

of the molecules reach equal

Thus the molecule transfers energy


.

The number of collisions of the molecules with the walls of surface S at time
equals
(1)
where n is the concentration of molecules and
, The mean velocity projection module of the molecules on the X axis
perpendicular to the wall. To make the assessment that you can take:
The mean square velocity v is determined by the formula

where M is the molar mass of the gas and T, the average temperature in the vial.
Since the values of T 1 and T 2 are coming we can take:
.
From the main equation of the kinetic theory of gases we obtain
.
Rewriting the formula (1):

meaning that at time

energy is transferred
(2).

For

content in the tea thermos cools from the temperature


until the temperature
energy must be transferred
.

Substituting this expression for W in the formula (2), we find the time
necessary to cool the tea:

REGIONAL PEACE

1st Secondary
1)

Conceptual part
List several repetitive phenomena that occur in nature and that could serve as reasonable
weather patterns.

2)
a.

You can combine two vectors of different magnitude to give a zero resultant?

b.

They can do three vectors?

3)

List several scalar quantities.

4)

Is time a vector? Why?

1)

Practical part
Express your height in miles. In your answer using only prefixes.

A plane travels 876.5 [km] in a straight line at an angle of 37.5 to the North

2)

East. How much the plane has traveled both to the North and to the East, from its starting
point? . Help:
3)

Two trucks carrying sand. One of these, B, has a small hole in its base through which the
sand is falling to the floor. They have taken the following mass versus time for both trucks:
Data #
Time [min]
A forklift Mass [kg]
Cart B mass [kg]
1
0.0
8.75
8.75
2
1.0
8.76
7.77
3
2.0
8.75
6.74
4
3.0
8.77
5.76
5
4.0
8.75
4.75
6
5.0
8.75
3.76
7
6.0
8.74
2.73
a) Plot:

The mass versus time for the truck to

The mass versus time for the truck B

b) Because if A mass is constant we have different data?


The number of seconds in a year can be replaced conveniently by 10. What is the

4)

percentage error of this figure?. Take = 3.141592653


June 8, 2003

REGIONAL PEACE

2nd of Secondary
Conceptual part
1)
b.

2)

If a b = 0, does it follow that a and b are perpendicular to each other?

c.

If a = 0, Should be parallel to and b?

d.

Can a dot product between vectors be a negative amount?


List several scalar and vector quantities.

3)
4)

What experimental evidence exists to assume that the speed of sound is the same for all
wavelengths?
What is it that causes mirages? Do you have something to do with the fact that the refractive

1)

index of air is not constant but changes to its density? Draw rays explaining the phenomenon.
Practical part
A cylindrical vessel containing water has a radius of 2 cm. In 2 hours the water level low 1
mm. Estimate in grams per second, the evaporation rate at which water is evaporating.

2)

The lowest tone that can detect the human ear as sound is about 20 [Hz] and the highest is
about 20000 [Hz]. What is the wavelength of each of them in the air?. The speed of sound in
air is 331.3 [ms -1]. Indicates a wavelength where the wavelength and amplitude.

3)

What should be the height of a vertical mirror, so that a person 6 feet tall to be able to
observe the entire image?. Assume that your eyes are 4 inches below the top is its head

4)

Has been taken following the motion of a body:


Data #
Time [s]
Position [m]
Speed [ms -1]

Acceleration [ms 2]

1
2
3
4
5
6

0.0
1.0
2.0
2.5
3.5
4.0

1.0
3.7
4.8
5.0
3.9
1.4

4.0
1.8
0.7
0.0
-3.0
-6.2

-3.8
-1.2
-1.0
-1.9
-4.9
-7.3

Plot:

the position versus time

the speed versus time

acceleration versus time.


June 8, 2003

REGIONAL PEACE

3rd of Secondary
SOLUTIONS
1)

Conceptual part
What is the difference between mass and weight? Much do you weigh?

2)

Everyone knows that when you look in a mirror is an inversion of the left and right. The
right hand seems to be a left, left eye seems to be one law, etc.. Is it possible to design a mirror
system that would allow to observe an image as the way in which people normally
observed? If possible, draw a picture with some typical ray showing that this is true.

3)
4)

Explain: Archimedes Principle, equations of continuity and Bernoulli,


As you may know the distance that has been struck by lightning from where you find
yourself?
Practical part

1)
a.

Using the definition of scalar product a b = (a) (b) cos and that
a b = a x b x + a and b and b z + z obtaining the angle between the vectors:
a = 3 i + 3 j - 3 k and b = 2 i + 3 k

b.
2)

Plot the vectors a = 3 i + 3 j + 3 k, and b = 2 i + 3 k and the vector product a


He drops a stone from the top of a building. The sound of the stone to hit the ground is

heard 6.5 s later. If the speed of sound is 1120 ft s -1 Calculate the height of the building.

3)

(Venturi tube) analytically determining the fluid velocity, density known, it travels
through the tube shown in the figure, where MP1 and MP2 are two pressure
gauges. A 1 and A 2 are the areas perpendicular to the flow in regions 1 and 2 respectively.

Finally with data:


= 1 g / cc, p 1 = 1000 N m -2 , P 2 = 800 N m -2, A 1 = 40 cm 2, A 2 = 22 cm 2, calculate v 1
June 8, 2003

REGIONAL PEACE

4th of Secondary
1)
2)
3)

Conceptual part
Explain the Steiner theorem, also known as the parallel axis theorem.
Explain that the dynamic lifting force is the force acting on a body because of its motion
through a fluid.
Comment: 1st and 2nd law of thermodynamics, entropy, reversible and irreversible
processes, isothermal processes and adiabatic.

4)

Explain what the Doppler effect.

1)

Practical part
A disc of 0.5 [m] radius and 20 [kg] of mass can rotate freely about a fixed horizontal axis
passing through its center. Applies a force F of 9.8 [N] pulling a cord coiled around the edge of
the disc.Find the angular acceleration of the disk and its angular velocity after 2 [s]. Finding the
reaction force also exists in the brackets which support the axis of the disc.

2)

A semiquantitative definition of electric flux is:


us that: q
sphere
vectors

3)

. And Gauss's law tells

. Write the electric field of a point charge q taking Gaussian surface as a


of

radius r. Help:
and

The

total

area

of

sphere

is

r 2 and

the

have the same direction.

A boat in motion produces surface waves on a calm lake. The boat runs 12 oscillations in 20
[s]; each oscillation produces a wave crest. The crest of the wave takes 6 [s] to reach the distant
shore 12 [m]. Calculate the wavelength of surface waves.

4)

Three capacitors of 1.5 F], 2 F] and 3 F] are connected in a) series, b) are parallel
and applies a potential difference of 20 [V]. Determine in each case i) the capacitance of the
system, ii) loading and the potential difference of each capacitor, iii) energy of the system.

REGIONAL PEACE

1st Secondary
SOLUTIONS
1)

Conceptual part
List several repetitive phenomena that occur in nature and that could serve as reasonable
weather patterns.

Sun The time when the moon becomes full, the beginning of a season, solar cycles, the
movement of the stars, etc..
2)
a.

You can combine two vectors of different magnitude to give a zero resultant?

Sun b.

They can do three vectors?

Sun 3)
4)

1)
2)

No
If

List several scalar quantities.


Sun - mass, time, density, volume, temperature, electric current, etc..
Is time a vector? Why?
Sun - No, that the management and direction of a vector has no restriction on its orientation.
Practical part
Express your height in miles. In your answer using only prefixes.
Sun - If you measure 1.67 meters then say you write 1.67 milli Kilo meters.
A plane travels 876.5 [km] in a straight line at an angle of 37.5 to the North East. How
much the plane has traveled both to the North and to the East, from its starting point? . Help:

Sun -

Eastbound:

876.5 cos (52.5 = 533.6 [km]

Northbound

876.5 sin (52.5 = 695.4 [km]

Since 90 - 37.5 = 52.5 angle that is taken with respect to the horizontal.
3)

Two trucks carrying sand. One of these, B, has a small hole in its base through which the
sand is falling to the floor. They have taken the following mass versus time for both trucks:
Data #
Time [min]
A forklift Mass [kg]
Cart B mass [kg]
1
0.0
8.75
8.75
2
1.0
8.76
7.77
3
2.0
8.75
6.74
4
3.0
8.77
5.76
5
4.0
8.75
4.75
6
5.0
8.75
3.76
7
6.0
8.74
2.73
a) Plot:

The mass versus time for the truck to

The mass versus time for the truck B

b) Because if A mass is constant we have different data?


Sun - Because when we take measurements random errors always exist, why not
find a single result if not several. But note that the difference between them
(mass of Cart A) is not great. These differences are known as natural statistical
fluctuations.
The number of seconds in a year can be replaced conveniently by 10. What is the

4)

percentage error of this figure?. Take = 3.141592653


Sun - 10 = 31415926.53
In a year of 365 days or 8760 hours are 31,536,000 seconds. Therefore the error percentage is
0.38%
Remember
And
This is a rough estimate because in a year when there are actually 365.2 days.
June 8, 2003

REGIONAL PEACE

2nd of Secondary
SOLUTIONS
Conceptual part
5)
a.

If a b = 0, does it follow that a and b are perpendicular to each other?

Sun - Yes, since by definition a b = (a) (b) cos as cos = 0 if and only if =
90 then a and b are perpendicular.
b.

If a = 0, Should be parallel to and b?

Sun - Yes, since by definition a = (a) (b) sin u and as sin = 0 if and only if =
0 then a and b are parallel.
c.

Can a dot product between vectors be a negative amount?

Sun - Yes, you can. Which can not be negative is the magnitude of a vector.
6)

List several scalar and vector quantities.


Sun -

7)

8)

Scalar: mass, time, density, volume, temperature, electric current, etc..


Vectors: Strength, Speed, Acceleration, Magnetic Fields, Electric, Gravity; Momentum,
Torque,
What experimental evidence exists to assume that the speed of sound is the same for all
wavelengths?
Sun - Any piece of music, which is a collection of different wavelengths of sound waves,
could never be heard as we do if the speed was different for certain wavelengths, as certain
notes come to us much later or earlier than the other, you think? Would hear at the wrong
without understanding anything.
What is it that causes mirages? Do you have something to do with the fact that the refractive
index of air is not constant but changes to its density? Draw rays explaining the phenomenon.
Sun - They did have a lot to do. The refractive index is sensitive to temperature
changes. The mirages usually happen in deserts or on paved roads in a region very hot places
exceeding 40 C, so ground-level air is hot and have lower density than the cooler air rises

5)

creating a series of strata or layers of different temperature levels. A layer of cold air, another
of hot air and cold another produce a lens which bends the light rays and makes an image
appear which can not be.
Practical part
A cylindrical vessel containing water has a radius of 2 cm. In 2 hours the water level low 1
mm. Estimate in grams per second, the evaporation rate at which water is evaporating.
[Cc s -1]. 1cc water = 1 g of water. 1 [mm]

Sun - The volume of water that falls is


= 0.1 [cm]. Therefore
6)

[G s -1]

The lowest tone that can detect the human ear as sound is about 20 [Hz] and the highest is
about 20000 [Hz]. What is the wavelength of each of them in the air?. The speed of sound in
air is 331.3 [ms -1]. Indicates a wavelength where the wavelength and amplitude.
Sol - As
[mm]

and

[Ms -1] then the wavelengths are 16,565 [m] and 16,565

7)

What should be the height of a vertical mirror, so that a person 6 feet tall to be able to
observe the entire image?. Assume that your eyes are 4 inches below the top is its head
Sun - The figure shows the trajectories of rays that leave
the top of the head of the individual and his toes. These
rays chosen so that enter the eye after reflection, affect
the vertical mirror in points a and b respectively. The
mirror needs to occupy only that region between these two
points. Projecting rays reaching the eye towards the place
where they form the mirror image is that this should have
half the height of the individual, ie 3 feet. Note that this
height is independent of the distance at which is the person
in the mirror.

8)

Has been taken following the motion of a body:


Data #
Time [s]
Position [m]
Speed [ms -1]

Acceleration [ms 2]

1
2
3
4
5
6

0.0
1.0
2.0
2.5
3.5
4.0

1.0
3.7
4.8
5.0
3.9
1.4

Plot:

the position versus time

the speed versus time

acceleration versus time.

4.0
1.8
0.7
0.0
-3.0
-6.2

-3.8
-1.2
-1.0
-1.9
-4.9
-7.3

June 8, 2003

REGIONAL PEACE

3rd of Secondary
SOLUTIONS
1)

Conceptual part
What is the difference between mass and weight? Much do you weigh?
Sun - The mass is a scalar, its units are [kg], the weight is a vector, is a force therefore is
measured in Newtons [N]. If you say you have a mass of 66 [kg] weights then 66
[kg] [ms -2] = 645.15 [N].
9775 [ms -2] is gravity in La Paz.
9.81 [ms -2] is the gravity at sea level.
1.67 [ms -2] is the gravity on the moon. How much would you weigh on the moon?

2)

Everyone knows that when you look in a mirror is an inversion of the left and right. The
right hand seems to be a left, left eye seems to be one law, etc.. Is it possible to design a mirror
system that would allow to observe an image as the way in which people normally
observed? If possible, draw a picture with some typical ray showing that this is true.
Sun - If possible, be requires two mirrors that are joined at an angle of 90 between

3)

them. Each image in a mirror will be reflected in the other mirror as to achieve reverse what
we usually see as a result finding the desired image.
Explain: Archimedes Principle, equations of continuity and Bernoulli,

Sol - A body wholly or partly immersed in a fluid is pushed upward by a force that equals the
weight of the fluid displaced by said body (Archimedes).
The principle that expresses the conservation of mass in a fluid movement is known as the

Where is the fluid density, is the area


through which the fluid at a given time and
is the fluid
velocity. That the product of these three quantities remain constant is
a fancy way of saying that the mass does not change in the fluid.
continuity equation:

The principle that expresses the conservation of energy in the movement of a fluid is known as

4)

Bernoulli's equation:

Where

and

and

the height at which the fluid moves,

is the pressure,

the severity

continue to express the density and

velocity of fluid. The first term is the kinetic energy per unit volume [J m -3], the second and
third terms express the potential energy per unit volume [N m -2 = J m -3] due to internal forces
(pressure) and external forces respectively. Therefore the total energy per unit volume remains
constant.
As you may know the distance that has been struck by lightning from where you find
yourself?
Sun - First "see" the beam. Light travels at a speed of approximately

[Ms -1] so we

can say that lightning emits light reaches us instantly. Then we have the time

that comes to

us in the "sound" of the beam. The speed of sound in air is worth

[Ms -1]. Then

using the relationship

the distance found.


Practical part

1)
a.

Using the definition of scalar product a b = (a) (b) cos and that
a b = a x b x + a and b and b z + z obtaining the angle between the vectors:
a = 3 i + 3 j - 3 k and b = 2 i + 3 k

Sun - The angle will


b.
Sun -

2)

Plot the vectors a = 3 i + 3 j + 3 k, and b = 2 i + 3 k and the vector product a


a = 9i - 3 j - 6 k, which must be stressed is that both a and b form a

plane

in space which is perpendicular to the vector to the graph is easily accomplished by


locating the points in space (3,3,3), (2,0,2) and (9, -3, -6 ) indicating the end of each vector
starting at the origin (0,0,0).
He drops a stone from the top of a building. The sound of the stone to hit the ground is
heard 6.5 s later. If the speed of sound is 1120 ft s -1 Calculate the height of the building.
Sun - The total displacement covered by the stone and the sound is:

where g = -32.2 [ft / s 2]


also

then

[S]

where

solution:
3)

cleared

[S]. Finally

staying

only

with

the

positive

[Feet].

(Venturi tube) analytically determining the fluid velocity, density known, it travels
through the tube shown in the figure, where MP1 and MP2 are two pressure
gauges. A 1 and A 2 are the areas perpendicular to the flow in regions 1 and 2 respectively.

Finally with data:


= 1 g / cc, p 1 = 1000 N m -2 , P 2 = 800 N m -2, A 1 = 40 cm 2, A 2 = 22 cm 2, calculate v 1
Sun - From the continuity equation:

(A)
And the Bernoulli equation when the system is in a horizontal position:
(B)

If we replace (A) in (B) and then solve for

we obtain:

Replacing the data provided we find that:

13.16 [ms -1]

4)

Verify that when a wave hits a certain angle and passes through half limited by flat faces
parallel, the direction of propagation of the beam emerging is parallel to the incident beam.
Sun Consider the following figure:

Using the law of refraction or Snell's law we find:


Similarly:

n 1 sin i) = n 2 sin t)

n 2 sin = n 2 sin

Now within the medium 2 is true also that i =

t.

Combining these equations is reached t = i, which is what we wanted to prove.


June 8, 2003

REGIONAL PEACE
CONTEST II

4th of Secondary
SOLUTIONS
5)

6)

7)

Conceptual part
Explain the Steiner theorem, also known as the parallel axis theorem.
Sun - The parallel axis theorem is given by the relationship:
, Where I is the
inertia around the axis of rotation arbitrary I cm is the rotational inertia parallel axis passing
through the center of mass, M is the total mass of the object and h is the perpendicular distance
between the axes. The two axes are parallel. The moments of inertia with respect to the
parallel axes relate in this simple way.
Explain that the dynamic lifting force is the force acting on a body because of its motion
through a fluid.
Sun - These bodies are the wings of an airplane or a helicopter propellers. The streamlines of
air around the wing of an aircraft for example cause the air speed above the wing has a greater
magnitude than the air velocity below this, which means that the pressure above the wing is
less than pressure below which must occur if there is to be a push upward.
Comment: 1st and 2nd law of thermodynamics, entropy, reversible and irreversible
processes, isothermal processes and adiabatic.
Sun All thermodynamic system in an equilibrium state has a state variable, called the internal
energy of the system is such that its change

where

Energy is added to

is the energy supplied by the system when


performing work. (1st Law)
the system by heat transfer and

It is impossible for a cycler not produce another effect that continuously transfer heat from one
body to another at a higher temperature than this.
It is also possible to reformulate the 2nd law in terms of entropy: A natural process that begins
in an equilibrium state and ends in another place at the direction that causes the entropy of the
system and its environment increases.
Entropy is a thermodynamic variable that measures the disorder.
If a change occurs very slowly so that at each moment the system is in an equilibrium position
then the process is reversible. And if the system away from its equilibrium state then the
process is irreversible.
When there is a change in system pressure or volume but the temperature is kept constant is
spoken of an isothermal process.
When there are non-heat exchange between the system and the environment comes to an
adiabatic process. An adiabatic process can be reversible or irreversible.
8)

Explain what the Doppler effect.


When a person hears a sound is moving toward the stationary source that produces it, the
frequency of the sound heard is greater than when at rest. If the person is moving away from
the stationary source, less frequently heard when this than when at rest. Similarly is the

5)

equivalent when both source and person move away from or towards. This effect applies to all
waves in general.
Practical part
A disc of 0.5 [m] radius and 20 [kg] of mass can rotate freely about a fixed horizontal axis
passing through its center. Applies a force F of 9.8 [N] pulling a cord coiled around the edge of

the disc.Find the angular acceleration of the disk and its angular velocity after 2 [s]. Finding the
reaction force also exists in the brackets which support the axis of the disc.
Sun 6)

A semiquantitative definition of electric flux is:


us that: q
sphere

7)

. Write the electric field of a point charge q taking Gaussian surface as a


of

vectors

. And Gauss's law tells

radius r. Help:
and

The

total

area

of

sphere

r 2 and

is

the

have the same direction.

A boat in motion produces surface waves on a calm lake. The boat runs 12 oscillations in 20
[s]; each oscillation produces a wave crest. The crest of the wave takes 6 [s] to reach the distant
shore 12 [m]. Calculate the wavelength of surface waves.

8)

Three capacitors of 1.5 F], 2 F] and 3 F] are connected in a) series, b) are parallel
and applies a potential difference of 20 [V]. Determine in each case i) the capacitance of the
system, ii) loading and the potential difference of each capacitor, iii) energy of the system.

REGIONAL PEACE

3rd of Secondary
SOLUTIONS
1)

Conceptual part
What is the difference between mass and weight? Much do you weigh?
Sun - The mass is a scalar, its units are [kg], the weight is a vector, is a force therefore is
measured in Newtons [N]. If you say you have a mass of 66 [kg] weights then 66
[kg] [ms -2] = 645.15 [N].
9775 [ms -2] is gravity in La Paz.
9.81 [ms -2] is the gravity at sea level.
1.67 [ms -2] is the gravity on the moon. How much would you weigh on the moon?

2)

Everyone knows that when you look in a mirror is an inversion of the left and right. The
right hand seems to be a left, left eye seems to be one law, etc.. Is it possible to design a mirror
system that would allow to observe an image as the way in which people normally
observed? If possible, draw a picture with some typical ray showing that this is true.
Sun - If possible, be requires two mirrors that are joined at an angle of 90 between

3)

them. Each image in a mirror will be reflected in the other mirror as to achieve reverse what
we usually see as a result finding the desired image.
Explain: Archimedes Principle, equations of continuity and Bernoulli,
Sol - A body wholly or partly immersed in a fluid is pushed upward by a force that equals the
weight of the fluid displaced by said body (Archimedes).
The principle that expresses the conservation of mass in a fluid movement is known as the

Where is the fluid density, is the area


through which the fluid at a given time and
is the fluid
velocity. That the product of these three quantities remain constant is
a fancy way of saying that the mass does not change in the fluid.
continuity equation:

The principle that expresses the conservation of energy in the movement of a fluid is known as
Bernoulli's equation:

Where

and

and

the height at which the fluid moves,

is the pressure,

the severity

continue to express the density and

velocity of fluid. The first term is the kinetic energy per unit volume [J m -3], the second and
third terms express the potential energy per unit volume [N m -2 = J m -3] due to internal forces

4)

(pressure) and external forces respectively. Therefore the total energy per unit volume remains
constant.
As you may know the distance that has been struck by lightning from where you find
yourself?
Sun - First "see" the beam. Light travels at a speed of approximately

[Ms -1] so we

can say that lightning emits light reaches us instantly. Then we have the time

that comes to

us in the "sound" of the beam. The speed of sound in air is worth

[Ms -1]. Then

using the relationship

the distance found.


Practical part

1)
a.

Using the definition of scalar product a b = (a) (b) cos and that
a b = a x b x + a and b and b z + z obtaining the angle between the vectors:
a = 3 i + 3 j - 3 k and b = 2 i + 3 k

Sun - The angle will


b.

Plot the vectors a = 3 i + 3 j + 3 k, and b = 2 i + 3 k and the vector product a

Sun -

2)

a = 9i - 3 j - 6 k, which must be stressed is that both a and b form a

plane

in space which is perpendicular to the vector to the graph is easily accomplished by


locating the points in space (3,3,3), (2,0,2) and (9, -3, -6 ) indicating the end of each vector
starting at the origin (0,0,0).
He drops a stone from the top of a building. The sound of the stone to hit the ground is
heard 6.5 s later. If the speed of sound is 1120 ft s -1 Calculate the height of the building.
Sun - The total displacement covered by the stone and the sound is:

where g = -32.2 [ft / s 2]


also

[S]

then

where

solution:
3)

[S]. Finally

cleared

staying

only

with

the

positive

[Feet].

(Venturi tube) analytically determining the fluid velocity, density known, it travels
through the tube shown in the figure, where MP1 and MP2 are two pressure
gauges. A 1 and A 2 are the areas perpendicular to the flow in regions 1 and 2 respectively.

Finally with data:


= 1 g / cc, p 1 = 1000 N m -2 , P 2 = 800 N m -2, A 1 = 40 cm 2, A 2 = 22 cm 2, calculate v 1
Sun - From the continuity equation:

(A)
And the Bernoulli equation when the system is in a horizontal position:
(B)

If we replace (A) in (B) and then solve for

we obtain:

Replacing the data provided we find that:

13.16 [ms -1]

4)

Verify that when a wave hits a certain angle and passes through half limited by flat faces
parallel, the direction of propagation of the beam emerging is parallel to the incident beam.
Sun Consider the following figure:

Using the law of refraction or Snell's law we find:


Similarly:

n 1 sin i) = n 2 sin t)

n 2 sin = n 2 sin

Now within the medium 2 is true also that i =

t.

Combining these equations is reached t = i, which is what we wanted to prove.


June 8, 2003

REGIONAL PEACE
CONTEST II

4th of Secondary
SOLUTIONS
5)

Conceptual part
Explain the Steiner theorem, also known as the parallel axis theorem.
Sun - The parallel axis theorem is given by the relationship:
, Where I is the
inertia around the axis of rotation arbitrary I cm is the rotational inertia parallel axis passing
through the center of mass, M is the total mass of the object and h is the perpendicular distance

6)

7)

between the axes. The two axes are parallel. The moments of inertia with respect to the
parallel axes relate in this simple way.
Explain that the dynamic lifting force is the force acting on a body because of its motion
through a fluid.
Sun - These bodies are the wings of an airplane or a helicopter propellers. The streamlines of
air around the wing of an aircraft for example cause the air speed above the wing has a greater
magnitude than the air velocity below this, which means that the pressure above the wing is
less than pressure below which must occur if there is to be a push upward.
Comment: 1st and 2nd law of thermodynamics, entropy, reversible and irreversible
processes, isothermal processes and adiabatic.
Sun All thermodynamic system in an equilibrium state has a state variable, called the internal
energy of the system is such that its change

where

Energy is added to

is the energy supplied by the system when


performing work. (1st Law)
the system by heat transfer and

It is impossible for a cycler not produce another effect that continuously transfer heat from one
body to another at a higher temperature than this.
It is also possible to reformulate the 2nd law in terms of entropy: A natural process that begins
in an equilibrium state and ends in another place at the direction that causes the entropy of the
system and its environment increases.
Entropy is a thermodynamic variable that measures the disorder.
If a change occurs very slowly so that at each moment the system is in an equilibrium position
then the process is reversible. And if the system away from its equilibrium state then the
process is irreversible.
When there is a change in system pressure or volume but the temperature is kept constant is
spoken of an isothermal process.
When there are non-heat exchange between the system and the environment comes to an
adiabatic process. An adiabatic process can be reversible or irreversible.
8)

Explain what the Doppler effect.


When a person hears a sound is moving toward the stationary source that produces it, the
frequency of the sound heard is greater than when at rest. If the person is moving away from
the stationary source, less frequently heard when this than when at rest. Similarly is the

5)

equivalent when both source and person move away from or towards. This effect applies to all
waves in general.
Practical part
A disc of 0.5 [m] radius and 20 [kg] of mass can rotate freely about a fixed horizontal axis
passing through its center. Applies a force F of 9.8 [N] pulling a cord coiled around the edge of
the disc.Find the angular acceleration of the disk and its angular velocity after 2 [s]. Finding the
reaction force also exists in the brackets which support the axis of the disc.
Sun -

6)

A semiquantitative definition of electric flux is:


us that: q
sphere
vectors

7)

. And Gauss's law tells

. Write the electric field of a point charge q taking Gaussian surface as a


of

radius r. Help:
and

The

total

area

of

sphere

is

r 2 and

the

have the same direction.

A boat in motion produces surface waves on a calm lake. The boat runs 12 oscillations in 20
[s]; each oscillation produces a wave crest. The crest of the wave takes 6 [s] to reach the distant
shore 12 [m]. Calculate the wavelength of surface waves.

Three capacitors of 1.5 F], 2 F] and 3 F] are connected in a) series, b) are parallel

8)

and applies a potential difference of 20 [V]. Determine in each case i) the capacitance of the
system, ii) loading and the potential difference of each capacitor, iii) energy of the system.

REGIONAL PEACE

3rd of Secondary
SOLUTIONS
1)

Conceptual part
What is the difference between mass and weight? Much do you weigh?
Sun - The mass is a scalar, its units are [kg], the weight is a vector, is a force therefore is
measured in Newtons [N]. If you say you have a mass of 66 [kg] weights then 66
[kg] [ms -2] = 645.15 [N].
9775 [ms -2] is gravity in La Paz.
9.81 [ms -2] is the gravity at sea level.
1.67 [ms -2] is the gravity on the moon. How much would you weigh on the moon?

2)

Everyone knows that when you look in a mirror is an inversion of the left and right. The
right hand seems to be a left, left eye seems to be one law, etc.. Is it possible to design a mirror
system that would allow to observe an image as the way in which people normally
observed? If possible, draw a picture with some typical ray showing that this is true.
Sun - If possible, be requires two mirrors that are joined at an angle of 90 between

3)

them. Each image in a mirror will be reflected in the other mirror as to achieve reverse what
we usually see as a result finding the desired image.
Explain: Archimedes Principle, equations of continuity and Bernoulli,
Sol - A body wholly or partly immersed in a fluid is pushed upward by a force that equals the
weight of the fluid displaced by said body (Archimedes).
The principle that expresses the conservation of mass in a fluid movement is known as the

Where is the fluid density, is the area


through which the fluid at a given time and
is the fluid
velocity. That the product of these three quantities remain constant is
a fancy way of saying that the mass does not change in the fluid.
continuity equation:

The principle that expresses the conservation of energy in the movement of a fluid is known as

4)

Bernoulli's equation:

Where

and

and

the height at which the fluid moves,

is the pressure,

the severity

continue to express the density and

velocity of fluid. The first term is the kinetic energy per unit volume [J m -3], the second and
third terms express the potential energy per unit volume [N m -2 = J m -3] due to internal forces
(pressure) and external forces respectively. Therefore the total energy per unit volume remains
constant.
As you may know the distance that has been struck by lightning from where you find
yourself?
Sun - First "see" the beam. Light travels at a speed of approximately

[Ms -1] so we

can say that lightning emits light reaches us instantly. Then we have the time

that comes to

us in the "sound" of the beam. The speed of sound in air is worth

[Ms -1]. Then

using the relationship

the distance found.


Practical part

1)
a.

Using the definition of scalar product a b = (a) (b) cos and that

a b = a x b x + a and b and b z + z obtaining the angle between the vectors:


a = 3 i + 3 j - 3 k and b = 2 i + 3 k

Sun - The angle will


b.

Plot the vectors a = 3 i + 3 j + 3 k, and b = 2 i + 3 k and the vector product a

Sun -

2)

a = 9i - 3 j - 6 k, which must be stressed is that both a and b form a

plane

in space which is perpendicular to the vector to the graph is easily accomplished by


locating the points in space (3,3,3), (2,0,2) and (9, -3, -6 ) indicating the end of each vector
starting at the origin (0,0,0).
He drops a stone from the top of a building. The sound of the stone to hit the ground is
heard 6.5 s later. If the speed of sound is 1120 ft s -1 Calculate the height of the building.
Sun - The total displacement covered by the stone and the sound is:

where g = -32.2 [ft / s 2]


also

[S]

then

where

solution:
3)

[S]. Finally

cleared

staying

only

with

the

positive

[Feet].

(Venturi tube) analytically determining the fluid velocity, density known, it travels
through the tube shown in the figure, where MP1 and MP2 are two pressure
gauges. A 1 and A 2 are the areas perpendicular to the flow in regions 1 and 2 respectively.

Finally with data:


= 1 g / cc, p 1 = 1000 N m -2 , P 2 = 800 N m -2, A 1 = 40 cm 2, A 2 = 22 cm 2, calculate v 1
Sun - From the continuity equation:

(A)
And the Bernoulli equation when the system is in a horizontal position:
(B)

If we replace (A) in (B) and then solve for

we obtain:

Replacing the data provided we find that:

13.16 [ms -1]

4)

Verify that when a wave hits a certain angle and passes through half limited by flat faces
parallel, the direction of propagation of the beam emerging is parallel to the incident beam.
Sun Consider the following figure:

Using the law of refraction or Snell's law we find:


Similarly:

n 1 sin i) = n 2 sin t)

n 2 sin = n 2 sin

Now within the medium 2 is true also that i =

t.

Combining these equations is reached t = i, which is what we wanted to prove.


June 8, 2003

REGIONAL PEACE
CONTEST II

4th of Secondary
SOLUTIONS
5)

6)

7)

Conceptual part
Explain the Steiner theorem, also known as the parallel axis theorem.
Sun - The parallel axis theorem is given by the relationship:
, Where I is the
inertia around the axis of rotation arbitrary I cm is the rotational inertia parallel axis passing
through the center of mass, M is the total mass of the object and h is the perpendicular distance
between the axes. The two axes are parallel. The moments of inertia with respect to the
parallel axes relate in this simple way.
Explain that the dynamic lifting force is the force acting on a body because of its motion
through a fluid.
Sun - These bodies are the wings of an airplane or a helicopter propellers. The streamlines of
air around the wing of an aircraft for example cause the air speed above the wing has a greater
magnitude than the air velocity below this, which means that the pressure above the wing is
less than pressure below which must occur if there is to be a push upward.
Comment: 1st and 2nd law of thermodynamics, entropy, reversible and irreversible
processes, isothermal processes and adiabatic.
Sun -

All thermodynamic system in an equilibrium state has a state variable, called the internal
energy of the system is such that its change

where

Energy is added to

is the energy supplied by the system when


performing work. (1st Law)
the system by heat transfer and

It is impossible for a cycler not produce another effect that continuously transfer heat from one
body to another at a higher temperature than this.
It is also possible to reformulate the 2nd law in terms of entropy: A natural process that begins
in an equilibrium state and ends in another place at the direction that causes the entropy of the
system and its environment increases.
Entropy is a thermodynamic variable that measures the disorder.
If a change occurs very slowly so that at each moment the system is in an equilibrium position
then the process is reversible. And if the system away from its equilibrium state then the
process is irreversible.
When there is a change in system pressure or volume but the temperature is kept constant is
spoken of an isothermal process.
When there are non-heat exchange between the system and the environment comes to an
adiabatic process. An adiabatic process can be reversible or irreversible.
8)

Explain what the Doppler effect.


When a person hears a sound is moving toward the stationary source that produces it, the
frequency of the sound heard is greater than when at rest. If the person is moving away from
the stationary source, less frequently heard when this than when at rest. Similarly is the

5)

equivalent when both source and person move away from or towards. This effect applies to all
waves in general.
Practical part
A disc of 0.5 [m] radius and 20 [kg] of mass can rotate freely about a fixed horizontal axis
passing through its center. Applies a force F of 9.8 [N] pulling a cord coiled around the edge of
the disc.Find the angular acceleration of the disk and its angular velocity after 2 [s]. Finding the
reaction force also exists in the brackets which support the axis of the disc.
Sun -

6)

A semiquantitative definition of electric flux is:


us that: q
sphere
vectors

7)

. And Gauss's law tells

. Write the electric field of a point charge q taking Gaussian surface as a


of

radius r. Help:
and

The

total

area

of

sphere

is

r 2 and

the

have the same direction.

A boat in motion produces surface waves on a calm lake. The boat runs 12 oscillations in 20
[s]; each oscillation produces a wave crest. The crest of the wave takes 6 [s] to reach the distant
shore 12 [m]. Calculate the wavelength of surface waves.

8)

Three capacitors of 1.5 F], 2 F] and 3 F] are connected in a) series, b) are parallel
and applies a potential difference of 20 [V]. Determine in each case i) the capacitance of the
system, ii) loading and the potential difference of each capacitor, iii) energy of the system.

REGIONAL PEACE

3rd of Secondary
SOLUTIONS
1)

Conceptual part
What is the difference between mass and weight? Much do you weigh?
Sun - The mass is a scalar, its units are [kg], the weight is a vector, is a force therefore is
measured in Newtons [N]. If you say you have a mass of 66 [kg] weights then 66
[kg] [ms -2] = 645.15 [N].

9775 [ms -2] is gravity in La Paz.


9.81 [ms -2] is the gravity at sea level.
1.67 [ms -2] is the gravity on the moon. How much would you weigh on the moon?
2)

Everyone knows that when you look in a mirror is an inversion of the left and right. The
right hand seems to be a left, left eye seems to be one law, etc.. Is it possible to design a mirror
system that would allow to observe an image as the way in which people normally
observed? If possible, draw a picture with some typical ray showing that this is true.
Sun - If possible, be requires two mirrors that are joined at an angle of 90 between

3)

them. Each image in a mirror will be reflected in the other mirror as to achieve reverse what
we usually see as a result finding the desired image.
Explain: Archimedes Principle, equations of continuity and Bernoulli,
Sol - A body wholly or partly immersed in a fluid is pushed upward by a force that equals the
weight of the fluid displaced by said body (Archimedes).
The principle that expresses the conservation of mass in a fluid movement is known as the

Where is the fluid density, is the area


through which the fluid at a given time and
is the fluid
velocity. That the product of these three quantities remain constant is
a fancy way of saying that the mass does not change in the fluid.
continuity equation:

The principle that expresses the conservation of energy in the movement of a fluid is known as

4)

Bernoulli's equation:

Where

and

and

the height at which the fluid moves,

is the pressure,

the severity

continue to express the density and

velocity of fluid. The first term is the kinetic energy per unit volume [J m -3], the second and
third terms express the potential energy per unit volume [N m -2 = J m -3] due to internal forces
(pressure) and external forces respectively. Therefore the total energy per unit volume remains
constant.
As you may know the distance that has been struck by lightning from where you find
yourself?
Sun - First "see" the beam. Light travels at a speed of approximately

[Ms -1] so we

can say that lightning emits light reaches us instantly. Then we have the time

that comes to

us in the "sound" of the beam. The speed of sound in air is worth

[Ms -1]. Then

using the relationship

the distance found.


Practical part

1)
a.

Using the definition of scalar product a b = (a) (b) cos and that
a b = a x b x + a and b and b z + z obtaining the angle between the vectors:
a = 3 i + 3 j - 3 k and b = 2 i + 3 k

Sun - The angle will


b.
Sun -

Plot the vectors a = 3 i + 3 j + 3 k, and b = 2 i + 3 k and the vector product a


a = 9i - 3 j - 6 k, which must be stressed is that both a and b form a

plane

in space which is perpendicular to the vector to the graph is easily accomplished by


locating the points in space (3,3,3), (2,0,2) and (9, -3, -6 ) indicating the end of each vector
starting at the origin (0,0,0).

2)

He drops a stone from the top of a building. The sound of the stone to hit the ground is
heard 6.5 s later. If the speed of sound is 1120 ft s -1 Calculate the height of the building.
Sun - The total displacement covered by the stone and the sound is:

where g = -32.2 [ft / s 2]


also

[S]

then

where

solution:
3)

cleared

[S]. Finally

staying

only

with

the

positive

[Feet].

(Venturi tube) analytically determining the fluid velocity, density known, it travels
through the tube shown in the figure, where MP1 and MP2 are two pressure
gauges. A 1 and A 2 are the areas perpendicular to the flow in regions 1 and 2 respectively.

Finally with data:


= 1 g / cc, p 1 = 1000 N m -2 , P 2 = 800 N m -2, A 1 = 40 cm 2, A 2 = 22 cm 2, calculate v 1
Sun - From the continuity equation:

(A)
And the Bernoulli equation when the system is in a horizontal position:
(B)

If we replace (A) in (B) and then solve for


Replacing the data provided we find that:
4)

we obtain:
13.16 [ms -1]

Verify that when a wave hits a certain angle and passes through half limited by flat faces
parallel, the direction of propagation of the beam emerging is parallel to the incident beam.
Sun Consider the following figure:

Using the law of refraction or Snell's law we find:


Similarly:

n 1 sin i) = n 2 sin t)

n 2 sin = n 2 sin

Now within the medium 2 is true also that i =

t.

Combining these equations is reached t = i, which is what we wanted to prove.


June 8, 2003

REGIONAL PEACE
CONTEST II

4th of Secondary
SOLUTIONS
5)

6)

7)

Conceptual part
Explain the Steiner theorem, also known as the parallel axis theorem.
Sun - The parallel axis theorem is given by the relationship:
, Where I is the
inertia around the axis of rotation arbitrary I cm is the rotational inertia parallel axis passing
through the center of mass, M is the total mass of the object and h is the perpendicular distance
between the axes. The two axes are parallel. The moments of inertia with respect to the
parallel axes relate in this simple way.
Explain that the dynamic lifting force is the force acting on a body because of its motion
through a fluid.
Sun - These bodies are the wings of an airplane or a helicopter propellers. The streamlines of
air around the wing of an aircraft for example cause the air speed above the wing has a greater
magnitude than the air velocity below this, which means that the pressure above the wing is
less than pressure below which must occur if there is to be a push upward.
Comment: 1st and 2nd law of thermodynamics, entropy, reversible and irreversible
processes, isothermal processes and adiabatic.
Sun All thermodynamic system in an equilibrium state has a state variable, called the internal
energy of the system is such that its change

where

Energy is added to

is the energy supplied by the system when


performing work. (1st Law)
the system by heat transfer and

It is impossible for a cycler not produce another effect that continuously transfer heat from one
body to another at a higher temperature than this.
It is also possible to reformulate the 2nd law in terms of entropy: A natural process that begins
in an equilibrium state and ends in another place at the direction that causes the entropy of the
system and its environment increases.
Entropy is a thermodynamic variable that measures the disorder.

If a change occurs very slowly so that at each moment the system is in an equilibrium position
then the process is reversible. And if the system away from its equilibrium state then the
process is irreversible.
When there is a change in system pressure or volume but the temperature is kept constant is
spoken of an isothermal process.
When there are non-heat exchange between the system and the environment comes to an
adiabatic process. An adiabatic process can be reversible or irreversible.
8)

Explain what the Doppler effect.


When a person hears a sound is moving toward the stationary source that produces it, the
frequency of the sound heard is greater than when at rest. If the person is moving away from
the stationary source, less frequently heard when this than when at rest. Similarly is the

5)

equivalent when both source and person move away from or towards. This effect applies to all
waves in general.
Practical part
A disc of 0.5 [m] radius and 20 [kg] of mass can rotate freely about a fixed horizontal axis
passing through its center. Applies a force F of 9.8 [N] pulling a cord coiled around the edge of
the disc.Find the angular acceleration of the disk and its angular velocity after 2 [s]. Finding the
reaction force also exists in the brackets which support the axis of the disc.
Sun -

6)

A semiquantitative definition of electric flux is:


us that: q
sphere
vectors

7)

. And Gauss's law tells

. Write the electric field of a point charge q taking Gaussian surface as a


of

radius r. Help:
and

The

total

area

of

sphere

is

r 2 and

the

have the same direction.

A boat in motion produces surface waves on a calm lake. The boat runs 12 oscillations in 20
[s]; each oscillation produces a wave crest. The crest of the wave takes 6 [s] to reach the distant
shore 12 [m]. Calculate the wavelength of surface waves.

8)

Three capacitors of 1.5 F], 2 F] and 3 F] are connected in a) series, b) are parallel
and applies a potential difference of 20 [V]. Determine in each case i) the capacitance of the
system, ii) loading and the potential difference of each capacitor, iii) energy of the system.

Potrebbero piacerti anche